Clinical Integration: Multiple Choice Questions

¡Supera tus tareas y exámenes ahora con Quizwiz!

A 52-year-old gentleman comes to your office with a history of intermittent difficulty swallowing solid food. His symptoms have been present for the past 5 years. He points to his supraclavicular notch when describing where the food feels stuck, although he is able to chew his food and transfer it into his posterior pharynx without difficulty. He does not choke or cough while eating. Drinking water will usually relieve his symptoms, although on several occasions he has self-induced vomiting. His symptoms are slightly worse now than they were several years ago, which prompted today's visit. His symptoms typically happen shortly after swallowing solid food, particularly when he is eating fast and takes a large bite of food such as bread or meat. He does not have difficulty swallowing liquids. The longest he has had food stuck has been 30 minutes, during which time he had severe pain in his chest. He reports rare episodes of heartburn and regurgitation, which have been relieved with antacids. He denies smoking or excessive alcohol use. What Is the Most Likely Diagnosis? A. Esophageal ring (ie, Schatzki ring) B. Esophageal cancer C. Achalasia D. Esophageal spasm E. Peptic stricture

Correct answer: A He has esophageal (rather than oropharyngeal) dysphagia. He has no difficulty with liquids, suggesting that his symptoms are most likely due to a mechanical obstruction rather than a motor disorder such as esophageal spasm or achalasia. He has had only occasional episodes of heartburn or regurgitation to suggest acid reflux disease, and therefore, although possible, it less likely that he has a peptic stricture of his distal esophagus. Likewise, esophageal cancer is less likely because his symptoms have been present for approximately 5 years and, despite this, he has not lost a significant amount of weight. Patients with esophageal rings usually have intermittent dysphagia to solids, particularly foods greater than 13 mm in diameter. The episodes of dysphagia are usually associated with chest discomfort and relieved by regurgitating the obstructing food bolus. Acute impaction of the esophagus may require endoscopic intervention to remove the food bolus.

A 41-year-old woman presents to your clinic complaining of a "red eye." The redness is diffuse and has been present for 2 days. She also reports mild, watery discharge and a slight sensation of grittiness. She does not have significant pain and has not noticed any change in her vision. You are reassured by the lack of pain or visual changes. Further questioning confirms the absence of photophobia or a foreign body sensation beyond slight grittiness. She does not take any medications, denies previous episodes, and does not have any other systemic complaints. She tells you that her 3-year-old daughter recently had a "cold." Which of the Following Is the Most Likely Cause of Her Red Eye? A. Viral conjunctivitis B. Bacterial conjunctivitis C. Scleritis D. Endophthalmitis

Correct answer: A The patient has viral conjunctivitis as suggested by the watery discharge, recent exposure to an ill family member, and lack of alarm symptoms or signs of systemic illness. Nonpurulent discharge is inconsistent with bacterial conjunctivitis.

A 56-year-old man presents to the emergency department (ED) with a complaint of chest pain that began 60 minutes earlier and has not resolved. He states he has never had a heart attack before. He is a current smoker and has smoked 1 pack per day for 30 years. He has been having intermittent episodes of chest pain off and on for the last 4 months, but today was the first time that the chest pain persisted prompting him to visit the ED. The patient initially noticed the chest pain a few months ago while walking or climbing stairs. These episodes would resolve a few minutes after stopping and resting. In the last month, he has noticed that less effort would bring on the pain and has even noticed it while sitting watching television. Today, he awoke from sleep with chest pain that did not go away and so he came to the ED. Today's pain is a diffuse precordial burning and pressure that radiates to both shoulders and arms. Prior episodes have been felt in his lower jaw. Right now he feels nauseous and as if he cannot quite catch his breath; this is something he has never felt previously. On further questioning, some time ago, he was told his cholesterol was high but he never sought further follow-up. He has been under a great deal of stress at work during the last few months. What Is the Most Likely Diagnosis? A. Acute coronary syndrome B. Pericarditis C. Pulmonary embolism D. Gastroesophageal reflux

Correct answer: A The patient presents with acute coronary syndrome. He describes a prodrome of typical exertional angina that has progressed to rest symptoms over the last month (unstable angina). On presentation, the pain is prolonged and exhibits several features consistent with acute myocardial ischemia including an oppressive nature, radiation to the arms and shoulders, and associated dyspnea. These alarm symptoms are occurring on the backdrop of a high probability for coronary artery disease (CAD)—a 56-year-old man with typical angina and risk factors of smoking and high cholesterol has an estimated CAD prevalence of greater than 90%. An electrocardiogram and cardiac biomarkers should be obtained without delay in this patient.

A 23-year-old young woman comes to your office with a rash. The rash started as red, round, pruritic bumps on the abdomen approximately 1 week before presentation and has spread. More recently, 1.5-cm round scaly patches and plaques have developed on the trunk and extremities. Your patient is a graduate student and has recently begun working out with a personal trainer at the gym. She is sexually active with 1 partner. She does not routinely use condoms but has no history of genital ulceration. She recalls no antecedent sore throat or illness. She has no sun sensitivity or joint pain. She is otherwise healthy and takes no medications. She has no family history of psoriasis or lupus. On further physical examination, you note no unusual findings of the oral mucosa, no involvement of the scalp or hair, no nail changes, and no lymphadenopathy. What Is the Most Likely Diagnosis? A. Tinea corporis B. Psoriasis C. Pityriasis rosea D. Subacute cutaneous lupus erythematosus E. Secondary syphilis

Correct answer: A This patient has tinea corporis, likely contracted at her gym. The diagnosis is suggested by the presence of an annular, scaly, pruritic, papulosquamous eruption and could be confirmed by the identification of fungal hyphae on a potassium hydroxide preparation of skin scrapings. The history including her social history, medical history, and lack of additional symptoms also tends to exclude alternate diagnoses. Psoriasis may present in a similar manner. The characteristic lesions are annular, scaly plaques. This eruption may be triggered by a streptococcal throat infection, tends to be less pruritic than tinea corporis, and often involves the extensor surfaces, the scalp, the umbilicus, and/or the nails. Pityriasis rosea is often preceded by a viral upper respiratory infection. Pityriasis rosea may or may not be pruritic, usually begins with a single herald patch, and tends to follow Langer lines, leading to a "Christmas tree" distribution pattern. Subacute cutaneous lupus erythematosus (SCLE) occurs most commonly in white females. Patients with SCLE may have systemic symptoms including arthralgias or fatigue. The lesions are commonly annular but occur in a photodistributed pattern and have little scale. Secondary syphilis occurs 2 to 10 weeks after the primary chancre. Its appearance may vary but often appears as symmetric pink to brown papules on the trunk and proximal extremities. In the case of your patient, a negative potassium hydroxide test would rule out tinea corporis. A skin biopsy would then have been helpful to identify the pathohistologic patterns of the other dermatoses in the differential diagnosis.

A 46-year-old woman comes to your office because she is concerned about heavy vaginal bleeding. Her menstrual cycle has not changed, but over the past year, she has noticed increasingly heavy periods, requiring many more pads than she previously used, and the passage of blood clots. Over the past 3 months, she has become very tired as well, prompting her visit today. The interval between her periods has not changed, nor has she had a month without bleeding. She describes increased pelvic discomfort and menstrual cramps with her menses. She is sexually active and monogamous with her husband who has had a vasectomy. She has had no abnormal vaginal discharge. She denies increased sweating, cold intolerance, and constipation but, as stated, endorses significant fatigue. Her only medications are vitamin D and calcium. She has noticed no other bleeding sites and no easy bruising. She has no "alarm" symptoms, such as acute abdominal pain, dizziness, syncope, or palpitations. Office staff obtains a pregnancy test, which is negative. What Is the Most Likely Diagnosis? A. Endometrial cancer B. Uterine fibroids C. Coagulopathy D. Hypothyroidism

Correct answer: B Although the patient's age places her in an age range that might be perimenopausal, she has no symptoms that suggest this, and her cycle is regular. Despite her age, because she is not menopausal, she must be considered to be of reproductive age. Her negative pregnancy test is critical even given the history of her husband's vasectomy. Her bleeding is regular in nature and associated with increased menstrual cramping and pelvic pain, making it ovulatory bleeding. Her history does not include any medications that might lead to menorrhagia, and her history does not suggest that medication use, an endocrine abnormality, or a coagulopathy plays a role. Although the diagnosis of fibroids would require confirmation with imaging, her history and symptoms point to the category of "uterine lesion or mass," and fibroids is the mostly likely diagnosis among these. Fibroids are common in women in their 40s and commonly cause the increased menstrual cramps she describes.

A 37-year-old woman comes to your clinic for "constipation," which she has had since childhood. She states that a work-up by her pediatrician turned up nothing. She reports intermittent, crampy abdominal pain that reaches 8 out of 10 in severity and is relieved "mostly" by a bowel movement. She has a bowel movement nearly every day, but her stools are usually hard. When the pain occurs, her stools are "really hard." She was recently laid off from her job and has noted an increase in her abdominal pain and hard stools. She has no alarm symptoms such as fever, weight loss, or rectal bleeding. She has a sense of incomplete evacuation more than half the time but does not have to use her fingers to assist with defecation. She denies history of sexual abuse. What Is the Most Likely Diagnosis? A. Inflammatory bowel disease B. Constipation-predominant irritable bowel syndrome C. Slow transit constipation D. Defecatory disorder

Correct answer: B Constipation-predominant irritable bowel syndrome is most likely because the abdominal pain is a significant feature and is relieved by bowel movements; the symptoms increase with psychological stress; and the stools change in consistency (and/or number of bowel movements) with onset of pain. This is a type of normal transit constipation. The absence of alarm symptoms makes inflammatory bowel disease less likely. Daily bowel movements and the presence of pain make slow transit constipation less likely, but the Bristol Stool Scale could be useful to confirm this—lower type has longer transit time. She neither passes soft stools nor has to use manual maneuvers to evacuate, so a defecatory disorder is less likely.

A 75-year-old man presents to your office for a routine office visit. During the visit, you inquire about hearing loss. He wasn't planning on discussing this but reports that he has trouble understating the television or when someone speaks in a whisper; his family is concerned that he doesn't hear as well as he used to. He hasn't been attending social events or family gatherings recently because he feels embarrassed having to ask people to repeat words or phrases. The patient's hearing loss has been gradual, and both ears are affected equally. He hasn't noticed ringing in his ears, dizziness, or disequilibrium. He denies a sudden worsening of his symptoms. He has the most trouble hearing in crowded restaurants or large gatherings. What Is the Most Likely Diagnosis? A. Ménière's disease B. Presbycusis C. Acoustic neuroma D. Multiple sclerosis

Correct answer: B He has gradual-onset, slowly progressive hearing loss that is bilateral. The most likely diagnosis is therefore presbycusis. Ménière's disease is typically accompanied by aural fullness, tinnitus, and vertigo. The hearing loss of acoustic neuroma is unilateral and asymmetric. Multiple sclerosis can occasionally cause sudden hearing loss and vertigo.

A 21-year-old woman is upset to report vaginal discharge. The discharge has been present for the past 10 days. She has never had this before and appears to be very anxious. She has also noted an odor that is particularly embarrassing; she hesitated to share this information. Her first intercourse was 3 weeks ago, and she is very worried about a sexually transmitted infection. She has been with only one partner who is male. She generally uses condoms, but on one occasion, the condom broke. The discharge is thin in consistency; she denies clumped discharge or itching. She has no dysuria. What Is the Most Likely Diagnosis? A. Candidiasis B. Bacterial vaginosis C. Trichomoniasis D. Psoriasis

Correct answer: B Her vaginal discharge is thin and not clumped. She denies itching. The combination of these findings suggests that she does not have candidal infection; bacterial vaginosis is more common than trichomoniasis, which is a relatively rare infection. Sexual activity is a risk factor for sexually transmitted infection and for bacterial vaginosis, although bacterial vaginosis is not a sexually transmitted disease. Although psoriasis is a common cause of vulvar dermatitis, it does not cause vaginal discharge.

A 36-year-old man comes to your office because of a persistent cough that has been bothering him for the past 3 months. His cough is dry and is more frequent during the evenings. He also notes frequent nasal congestion, especially when he is exposed to dusts and cold weather. He reports no hemoptysis, weight loss, wheezing, fever, or changes in his appetite. The patient often feels a dripping sensation in the back of this throat. Over-the-counter antihistamines provide partial relief of his symptoms. What Is the Most Likely Diagnosis? A. Asthma B. UACS C. GERD D. Viral infection

Correct answer: B In this case, upper airway cough syndrome (UACS) is the most likely diagnosis. He reported a dripping sensation in the back of his throat, and his symptoms responded to an antihistamine medication. UACS, previously known as "postnasal drip syndrome," includes a wide constellation of rhinosinus disorders. Allergic rhinitis is one of the most common disorders. Although his symptoms may mimic a viral infection, the duration of his cough argues against an infectious cause. Asthma and gastroesophageal reflux -disease (GERD) are very common causes of cough, but in this case, the patient denied any of the characteristic features of these conditions.

A 70-year-old man lives with his daughter. She is awakened one night and finds him wandering in the kitchen with the pots in disarray. He cannot explain clearly what he is doing, and she is worried because this is a dramatic change for him. She takes him to see you for immediate evaluation. On focused questioning, the daughter reports that the patient has fallen occasionally over the last few weeks. He has been more confused over the last few days and has had a few episodes of urinary incontinence. Although he has had some memory problems for a year or 2, he has been unable to dress himself or to tend to hygiene only in the last few days. He has also had some headaches over the last few days. He has not had any fever or any change in medications. What Is the Most Likely Diagnosis? A. Meningitis B. Subdural hematoma from trauma C. Unwitnessed seizure with postictal confusion D. Stroke

Correct answer: B The history of trauma in an elderly patient leading to confusion over days to weeks suggests a subdural hematoma. The patient likely also has an underlying dementia, which is both a risk factor for falls and for subdural hematomas. The "acute on chronic" nature of a worsening over days to weeks on a baseline of a gradual decline over months to years is highly suggestive of a delirium in the setting of a decreased cognitive reserve from dementia. The other causes are less likely; think first of the most likely cause and secondly of treatable causes of confusion. There is no history of fever, so meningitis is less likely. Seizures cause fluctuating symptoms and are of more rapid onset. Stroke is less likely in the absence of weakness but is still a possibility. In particular, a right parietal infarction or lobar small hemorrhage remains possible.

A 67-year-old man comes to your clinic for his annual appointment concerned about increasing shortness of breath. A year ago he was able to walk up the stairs to his apartment without difficulty, but now he has a hard time walking one block. He has a 70 pack-year smoking history, and several previous attempts to stop smoking have been unsuccessful. This patient has had slowly progressive dyspnea with exercise but no symptoms at rest. The chronicity of the patient's symptoms and his ability to engage in conversation reassure you that he does not require urgent intervention. He expresses frustration that he is unable to get a full breath when simply walking around his home. The dyspnea often worsens when he has a "cold," but he denies an acute increase in symptoms. There are no other triggers. When you ask about related symptoms, he describes a persistent cough productive of thick green sputum. The cough has been an irritating presence for the past several months. He denies fevers, chest pain, chest tightness, or orthopnea. What Is the Most Likely Diagnosis? A. Asthma B. Chronic obstructive pulmonary disease C. Congestive heart failure D. Pneumonia

Correct answer: B The most common causes of dyspnea are primary cardiac and pulmonary causes, anemia, deconditioning, and functional dyspnea. The patient describes a productive cough for greater than 3 months and an impressive smoking history; both have high likelihood ratios for chronic obstructive pulmonary disease (COPD). The patient's presentation is highly suggestive of the chronic bronchitis variant of COPD. However, congestive heart failure and COPD frequently occur together; thus a diagnosis of one does not exclude the other. Further evaluation for cardiac causes should be pursued because the patient has several coronary artery disease risk factors (smoking, male sex, and age over 60) and CHF may be contributing to the patient's dyspnea. The classic symptoms of asthma are dyspnea, wheezing, and cough, which overlap with those of COPD, making diagnostic distinction difficult. In this case, the patient's older age of onset makes asthma less likely. Pneumonia is unlikely based on the chronicity of symptoms and lack of systemic symptoms or signs of infection.

A 45-year-old man comes to your office with a 2-week history of left ear pain. The ear pain began shortly after an upper respiratory infection. He describes the pain as "a pressure" and also notes "crackling" in the ear. The patient reports that he cannot hear well with his left ear; he describes sounds as being "muffled" on the left but normal on the right. He has had no fever and feels well, apart from being frustrated by his ear problem. He tried an over-the-counter decongestant and this seemed to help somewhat. What Is the Most Likely Diagnosis? A. Acute otitis media B. Serous otitis C. TMJ dysfunction D. Temporal arteritis

Correct answer: B The patient has acute ear pain that is not severe and follows a recent upper respiratory infection. His impaired hearing and the "crackling" he describes are most consistent with serous otitis (eustachian tube dysfunction). Acute otitis media generally causes severe pain and is accompanied by fever. It may cause hearing loss but generally not "crackling" in the ear. TMJ dysfunction can cause chronic ear pain, frequently bilateral, and often worse with chewing. Patients may report clicking in the jaw. Temporal arteritis can also cause pain with chewing. It is a type of claudication: Pain occurs with prolonged chewing and goes away shortly after stopping chewing. It almost never occurs in patients younger than 50. It does not cause hearing loss. Serous otitis is a very common cause of ear pain in adults. It can be bilateral but is more frequently unilateral. It frequently develops after an upper respiratory infection or in the setting of seasonal allergies. Oral decongestants may be helpful, and the symptoms may take up to 12 weeks to resolve.

A 40-year-old woman presents with 3 months of vague abdominal discomfort. It is predominantly epigastric in location but is often accompanied by a sense of "fullness" that makes it difficult for her to eat. Over the past few weeks, she has also noted a burning sensation in her upper abdomen and reports increased abdominal bloating, as well as postprandial nausea. On further questioning, you learn that she does not drink, smoke, or take NSAIDs. Eating occasionally causes early satiety, but she generally eats 3 meals a day, and food does not worsen or alleviate the discomfort. The discomfort is intermittent and occurs on some days but not others. It is also worsened at times of stress. There has been no change in stool habits and no vomiting or hematemesis. She has no alarm symptoms such as weight loss, bleeding, anemia, or dysphagia. What Is the Most Likely Diagnosis? A. Gastric cancer B. Nonulcer or functional dyspepsia C. GERD D. Peptic ulcer disease

Correct answer: B The patient has classic symptoms of functional or nonulcer dyspepsia. Symptoms commonly include epigastric discomfort, early satiety, and postprandial nausea. These symptoms have significant overlap with those associated with other diagnoses; thus, it is important to probe for features that might make an organic etiology likely. Gastric malignancy is generally associated with the presence of alarm symptoms, including age over 45 to 55, weight loss, bleeding and/or iron deficiency, or dysphagia. GERD typically manifests as a burning sensation with radiation into the chest and rarely presents with early satiety. Peptic ulcer disease is frequently associated with tobacco, alcohol, or NSAID use, and the discomfort is typically affected by food intake and often associated with vomiting or other alarm signs listed above.

A 27-year-old woman comes to your office to discuss her "sick headaches," which started during high school. Her mother nudged her to see you. The headaches do not awaken her from sleep but can be disabling and occasionally require her to miss work. Sometimes she vomits during an attack. Over the past 6 months, her headaches have become more severe and frequent, prompting her visit today. Her headaches are similar in quality to the ones she has had since adolescence. They are now more severe and frequent, but the location, character, and associated symptoms have not changed. They are unilateral, throbbing, and associated with nausea and photophobia. They are usually worse during the first 2 to 3 days of her menses. You determine that this is an "old" headache. She has no alarm symptoms such as fever, disequilibrium, focal weakness, or neck stiffness. The detail with which she can provide a careful history suggests that her mental status is normal. On some occasions, her headaches are preceded by unusual zigzag flashing lights off to the right side of her visual field lasting 20 minutes or so. On further questioning, she has been under a great deal of stress, and her sleep schedule has been erratic. She has also been drinking up to 5 to 6 cups of coffee per day. What Is the Most Likely Diagnosis? A. Tension-type headache B. Migraine C. Brain tumor D. Cervicogenic headache

Correct answer: B The patient has old headaches (ie, they are the same in character as her usual headaches, albeit more severe). Therefore, the diagnosis for her headaches remains the same. Cervicogenic headache is generally a disease of middle-aged or older patients and is uncommon in young women. Occasionally, it can occur in younger patients after whiplash-type neck injuries. It is most commonly a burning or dull pain in the occiput and forehead. Brain tumor is a new headache. The character and description of her pain have not changed, as one would expect if a new source, such as brain tumor, were the cause of the headache. The unilateral, throbbing nature of the headaches is typical of migraine. The occasional visual aura and associated nausea confidently establish the diagnosis of migraine with great certainty. These 2 features most accurately distinguish migraine from tension-type headache. She has no alarm symptoms. The challenge is to determine why the headaches are more severe and frequent now. Her headaches are likely worse due to a change in her lifestyle including triggers of irregular sleep and excessive caffeine.

A 61-year-old woman comes to your office for intermittent dizziness for the past 2 weeks. At times, she misses work due to the dizziness. When she awakens in the morning, she states, "The entire room spins." Nausea accompanies the dizziness. The episodes last less than a minute. The patient has had 2 previous similar episodes of dizziness over the past year that resolved spontaneously. She is otherwise healthy and has no chronic medical conditions. She reports no associated headaches, hearing loss, or focal neurologic symptoms. The episodes are most severe when she rolls over in bed or gets out of bed. What Is the Most Likely Diagnosis? A. Postural hypotension B. Benign paroxysmal positional vertigo C. Posterior circulation ischemic infarct D. Disequilibrium

Correct answer: B These symptoms are typical of benign paroxysmal positional vertigo (BPPV). It is more common in people over the age of 60 years. In addition, similar episodes have occurred in the past with spontaneous resolution. The episodes last less than 1 minute and are most severe when the patient moves her head. If the episodes only occurred upon standing, then it would be more consistent with postural hypotension. She has no alarm symptoms, such as neurologic deficits, which would signal a serious diagnosis. Demonstration of normal gait and absence of any chronic medical diseases, such as Parkinson's disease, would make disequilibrium less likely.

A 75-year-old man is brought in by his daughter for progressive mainly unilateral tinnitus, hearing loss, and dizziness for the past year. About 3 weeks ago, he began experiencing unsteadiness and sustained a fall while walking up the stairs. The patient's tinnitus is constant and nonpulsatile and has been interfering with enjoyment of his favorite TV shows. It is low- to moderate-pitched, fairly loud, and occurs mostly on his right side. He has not experienced nausea, headaches, focal weakness, seizures, or constitutional symptoms. In the past, he worked for many years as an airline mechanic. He reports that his sense of balance is off and that there is a heaviness or "deadness" in his right ear. During the interview, he asks you to repeat your questions, turning his head to the left to hear you better. The physical examination is essentially normal, including the cardiac examination. There is no nystagmus. Tympanic membrane appears normal, and you cannot hear any additional sounds. Additional audiologic testing shows moderate bilateral high-frequency sensorineural hearing loss affecting the right ear much more than the left. What Is the Most Likely Diagnosis? A. Cerumen impaction B. Acoustic neuroma C. Otosclerosis D. Hearing loss from trauma (airline mechanic)

Correct answer: B This 75-year-old man has several reasons to have nonpulsatile tinnitus. He was subjected to chronic noise exposure as an airline mechanic, which typically causes bilateral high-frequency hearing loss and tinnitus. Otosclerosis causes bilateral tinnitus and progressive conductive hearing loss. Cerumen impaction is associated with unilateral hearing loss and tinnitus but would be obvious on physical examination (eg, wax obscuring the tympanic membrane). In this case, alarm features include the presence of tinnitus associated with progressive unilateral hearing loss. Additionally, the patient is experiencing disequilibrium and has fallen. Magnetic resonance imaging of the head revealed an eighth nerve mass, which on careful resection was a schwannoma, without invasion. The patient's hearing improved, and his tinnitus diminished to tolerable levels.

A 60-year-old gentleman presents to you with a 3-day history of constant perianal pain and bright red blood seen in the toilet bowl. This morning, he noticed a small swollen mass at his anus when washing in the shower. He experienced a similar episode of pain and bleeding several months ago, which he attributed to hemorrhoids, but his symptoms today are far more severe. The patient is otherwise healthy and has no history of a serious medical condition such as Crohn's disease or cancer. He has been in a monogamous relationship with his wife for almost 40 years and has never had a sexually transmitted infection. He describes his current pain as excruciating and constant throughout the day. He estimates the amount of blood in the toilet bowl to be about 1 tablespoon. He has not had any other drainage from the area and denies any recent fevers or chills. Upon further questioning, he endorses chronic constipation that seems to have worsened over the last few months, resulting in increased straining with defecation. What Is the Most Likely Diagnosis? A. Perianal abscess B. Thrombosed external hemorrhoid C. Anal fissure D. Anal cancer E. Rectal prolapse

Correct answer: B This patient has a thrombosed external hemorrhoid, the cardinal features of which include severe pain, a bluish lump at the rectum, and bleeding. External hemorrhoids are located distal to the dentate line, which is significant because the anal mucosa in this region is innervated by sensory nerve fibers. When hemorrhoidal vessels become engorged and there is stasis of blood, a thrombus can form, resulting in severe pain. Predisposing factors include activities that increase intra-abdominal pressure, such as straining from chronic constipation, diarrhea, or cough, and changes in habitus, such as obesity, pregnancy, and accumulation of ascites. When a patient presents with anorectal pain and bleeding, there are a number of other conditions that must be considered. Patients with rectal cancer may develop pain and bleeding. Fevers, weight loss, and fatigue each increase the index of suspicion. If the patient is engaging in anorectal intercourse without barrier protection, sexually transmitted proctitis will be higher on the differential diagnosis. In patients with Crohn's disease, fevers and purulent drainage would cause concern for a perianal abscess or fistula. Rectal prolapse is a swollen mass (ie, the rectal mucosa) protruding through the anus: Bleeding may occur from the edematous, friable tissue. Severe pain does not occur unless the rectum becomes strangulated. Lastly, an anal fissure is a tear in the anal mucosa that can cause severe pain with defecation and bleeding. Unlike a thrombosed external hemorrhoid, however, a tender lump is not palpable. No matter the history, a perianal inspection and digital examination are imperative in making the final diagnosis.

A 36-year-old woman presents to the emergency department with left-sided flank pain. The pain is severe, located just below her left ribs, and has been constant for the past 12 hours. It does not radiate or change with position. She also reports a fever to 101°F and general malaise. The patient reports that her symptoms started approximately 1 day after she noticed a burning sensation when urinating. Associated symptoms included nausea and vomiting. Past medical history includes an uncomplicated pregnancy and a history of urinary tract infections. She does not smoke and does not use intravenous drugs. What Is the Most Likely Diagnosis? A. Nephrolithiasis B. Pyelonephritis C. Musculoskeletal strain D. Renal infarct

Correct answer: B This patient is presenting with an infection of the upper urinary tract or pyelonephritis. Pyelonephritis is an infection that most often ascends from the lower urinary tract, which explains her initial complaint of dysuria. In addition to flank pain, pyelonephritis may be associated with high fever, malaise, nausea, and vomiting. The pain of pyelonephritis is often constant as opposed to the colicky pain that is classically associated with nephrolithiasis. Pain from musculoskeletal strain would not be associated with fever, nausea, and vomiting. The patient does not have any risk factors, such as atrial fibrillation, that would increase her risk for renal infarction.

A 49-year-old mother of 3 presents to your office with "high fever." She was in her usual state of health until 3 days ago when she noticed malaise. The following day she felt "hot" and noted sharp chest pain when she took a deep breath. That night her temperature was 102.5°F. She then developed shortness of breath and a dry cough and so presents to you today for evaluation. Her temperature has been as high as 103.0°F over the last 3 days. There is no exact pattern to the fever, but it usually resolves with antipyretics. She has had no rash or joint aches. She has had myalgias but no neck pain or confusion. She remains active with her 3 children but has not traveled. All 3 children had a similar illness last week without cough but with a mild rash that faded within a day. She has had some abdominal cramping and diarrhea that were also self-limited. Her cough has become more productive of yellow sputum today, but no blood was noted. Her chest pain continues to be painful and is worse with respiration and cough. She takes no medications other than the over-the-counter analgesics for her fever. Her family history is negative for history of venous thromboembolism or early miscarriage. What Is the Most Likely Diagnosis? A. Pulmonary embolism B. Community-acquired pneumonia C. Gastroenteritis D. Rheumatoid arthritis

Correct answer: B This patient is suffering from an acute febrile illness for 3 days without a discernible pattern to peaks in temperature. Her recent sick contacts and the acute nature of the illness both suggest an infectious etiology rather than an inflammatory condition such as rheumatoid arthritis. Pulmonary symptoms predominate and suggest a respiratory infection. Gastroenteritis is typically viral in etiology and usually presents with significant nausea, vomiting, and/or diarrhea as the main constellation of symptoms. Pulmonary embolism needs to be a consideration in any patient with pleuritic chest pain. She has few risk factors for this condition; she is not on oral contraceptives, has had no recent hospitalization or immobilization, and has no family history of clotting disorders. This patient has pneumonia; the constellation of fever, cough, and pleuritic pain are characteristic. When considering an infectious etiology for fever, look for symptoms that herald potential complications or signify a critical condition. This patient has no alarm symptoms to suggest meningitis (headache or neck pain), a CNS infection (altered mentation), or sepsis (dizziness, altered mentation.) Causative pathogens for pneumonia include bacteria and viruses and, less frequently, fungi or parasites. Treatment is often guideline driven and utilizes antibiotics, fluid, and oxygenation when needed.

A previously healthy 18-year-old woman is seen in your office complaining of a 3-day episode of "red urine," which has now resolved. She is frightened because this has never happened to her before. The patient also complains of severe pain and burning upon urination, and she is urinating more frequently than normal, up to 12 times per day. She and her boyfriend had sexual intercourse for the first time 1 week ago. What Is the Most Likely Diagnosis? A. IgA nephropathy B. Urinary tract infection (cystitis) C. Bladder cancer D. Ureteral calculus E. Goodpasture's disease

Correct answer: B This patient's age, the negative past medical history, and the association of her red urine with dysuria, urgency, and -frequency of urination strongly suggest that she has a urinary tract infection. With the additional history that her symptoms occurred after intercourse, she most likely has "honeymoon cystitis." Her symptoms are so characteristic that many physicians would treat her with empiric antibiotic therapy based on history alone, with the caveat that she should return in 7 to 10 days if her symptoms do not resolve. IgA nephropathy and Goodpasture's disease can cause gross hematuria (which may cause mild to moderate dysuria). However, neither IgA nephropathy nor Goodpasture's disease is accompanied by intense dysuria, urgency, or frequency. Although IgA nephropathy is the most common cause of glomerular hematuria, approximately 50% to 60% of all adult women report that they have had a urinary tract infection at some point. Ureteral calculi often present with severe lower abdominal pain, which was not present in this patient. Bladder cancer most often occurs in patients over the age of 50 and would be extremely unusual in a 17-year-old woman.

A 24-year-old woman presents to her primary care physician complaining of a deep, achy pain in her lower abdomen. When the pain began just over 3 months ago, she thought it was her usual menstrual cramps. She has had menstrual cramps since menarche at age 12. On several occasions, they were so severe that she had to miss school. Previously, the pain usually lasted for 3 to 4 days and was sometimes improved with ibuprofen. She decided to see the doctor because of persistence of the pain. Her pain is similar in quality and severity to her usual menstrual cramping; however, over the last 3 months, she has had cramps even when she is not menstruating. You learn that her periods have been getting steadily heavier over the last 5 years to the point that she now soaks through a pad every hour for the first 3 days of her periods. She has a 3-year-old daughter and had a normal vaginal delivery of another child a year ago. She has not been sexually active since her divorce 10 months ago. She denies pain with intercourse when she was sexually active. She does not have any fever or chills. On further questioning, you find out she recently remembered being abused sexually as a teenager and has just started seeing a counselor for therapy. What Is the Most Likely Diagnosis? A. Pelvic inflammatory disease B. Endometriosis C. Uterine fibroids D. Somatization

Correct answer: C She has dysmenorrhea complicated by noncyclic chronic pelvic pain. Excessive vaginal bleeding (menorrhagia) is typical of uterine fibroids or adenomyosis. Recent sexual inactivity and the absence of fever make pelvic inflammatory disease unlikely. Endometriosis is a common finding on laparoscopy in patients with noncyclic chronic pelvic pain. However, the absence of infertility and deep dyspareunia make endometriosis less likely to be the cause of this patient's symptoms. A history of childhood abuse and psychosocial stress are common in patients with chronic pelvic pain, and although comorbid functional and psychiatric disorders occur frequently, they do not cause menorrhagia.

A 40-year-old woman sees you for "lumps" in her breasts. She describes a history of fibrocystic disease and lumpy breasts but thinks that her lumps are becoming more noticeable. She is worried because her maternal aunt was recently diagnosed with breast cancer at age 68. The patient reveals that she has had bilateral, diffuse, lumpy breasts since her 20s. These lumps are more prominent and tender just before her menses. She has not noticed any discrete breast mass and is not taking any new medications. She has no other alarm symptoms, including skin changes, nipple complaints, or systemic complaints. She has no significant medical problems and has not had a prior breast biopsy. Her reproductive history includes menarche at age 12; she has never been pregnant. She has taken the same oral contraceptive for the last 10 years. Her family history is limited to her maternal aunt. She had a baseline mammogram 6 months ago, which was normal. What Is the Most Likely Diagnosis? A. Breast cancer B. Papilloma C. Fibrocystic disease D. Mastitis

Correct answer: C The most important aspect of her history is the lack of a discrete mass. The majority of breast complaints in the absence of a mass are due to benign causes. In some instances, the patient may feel that her breasts are so lumpy that she cannot tell if there is a discrete mass, and the examination will provide additional information. Moreover, the cyclical nature of her complaint supports the diagnosis of fibrocystic disease. Her breast cancer risk factors are minimal and include nulliparity and a modest family history in a second-degree relative. Her 5-year risk of developing breast cancer, based on the Gail Model, is 0.6% and identical to that of an average 40-year-old woman.

You receive a telephone call from a 32-year-old woman who complains of burning with urination over the past 12 hours. She also feels an intense need to urinate but reports difficulty voiding more than a small amount. She has experienced similar symptoms in the past and requests that you call in a prescription for antibiotics to her pharmacy. Your patient states that she had similar symptoms 6 months ago, which were diagnosed as a urinary tract infection. Her symptoms at that time resolved completely with antibiotic treatment. She denies any current vaginal irritation or discharge. She has had one male sexual partner for the past 2 years, does not use condoms, and has not had any sexually transmitted diseases. She denies fever, chills, nausea, vomiting, and back, pelvic, or abdominal pain. She does not have diabetes and is not pregnant. What Is the Most Likely Diagnosis? A. Pyelonephritis B. Urethrocystitis due to herpes simplex virus C. Lower urinary tract infection D. Vaginal candidiasis

Correct answer: C Your patient reports dysuria and urgency, suggesting a lower UTI. Given the prior UTI history, her pretest probability of an acute UTI is 85% to 90%. The absence of vaginal irritation or discharge makes vulvovaginitis less likely and increases the pretest probability of UTI to over 90%. Her sexual history indicates a relatively low risk for STIs, such as herpes or chlamydia urethritis. She does not endorse alarm symptoms suggesting pyelonephritis or pelvic inflammatory disease. Her high pretest probability of UTI and lack of risk factors for a complicated UTI suggest that urine culture is unnecessary. Thus she may be safely managed with telephone advice and empiric antibiotic treatment.

A 20-year-old college wrestler comes to your office with his mother because she is concerned that he has "passed out" in church. While attending Sunday services during the summer, he has twice become dizzy while standing and slumped to the floor. He was briefly "out of it" but revived quickly when taken outside. On each occasion, he insisted on completing the religious service. On both occasions, the church was crowded and hot, and the congregation had been standing for a prolonged period. He initially felt a graying out of his vision, then his hearing began to fade, and finally, he felt suddenly very hot and sweaty. The next thing he remembers was being surrounded by people fanning him as he came to consciousness. His mother reported that he slumped down to the floor very suddenly and looked "pale and clammy." He spontaneously recovered and was only momentarily confused. The other episode occurred the previous summer under similar circumstances. He is taking no medications, has no history of cardiac disease, and denies palpitations. There is no history of seizures. He keeps himself fit and exercises regularly to keep in shape for the wrestling season. There is no family history of sudden death, although a paternal grandfather died in his seventies of "heart attack." His mother and father are healthy. What Is the Most Likely Diagnosis? A. Sinus node dysfunction B. Long QT syndrome C. Seizure D. Vasovagal syncope E. Orthostatic hypotension

Correct answer: D Loss of consciousness is the sine qua non of syncope. The rapid onset with spontaneous, complete recovery excludes other forms of loss of consciousness such as coma, drug intoxication, and seizure. This is true syncope. The circumstances of a hot, crowded environment with prolonged standing and a previous history are characteristic of vasovagal syncope, as are the premonitory symptoms. The negative family and medical history and absence of palpitations also support that diagnosis. Orthostatic hypotension typically occurs with standing and has an underlying autonomic disorder or history compatible with volume loss.

A 26-year-old woman comes to clinic for evaluation of abnormal menses. After having previously irregular menses, she now reports a 9-month history of amenorrhea. She and her husband are now interested in having a child. The patient underwent menarche at age 13 and has always had irregular periods. In the last 3 years, her periods have become increasingly irregular with several episodes of heavy bleeding. She has also gained a significant amount of weight over this period. She denies headaches, vision changes, cold intolerance, changes in her hair or nails, hot flashes, or change in libido. She takes no medications. On physical examination, the patient is overweight with a body mass index of 29. Blood pressure is 122/68 mm Hg, and pulse 70 bpm. She has coarse hair noted on her upper lip and a few terminal hairs on her chin. Her abdomen is obese. Reflexes are normal. Pelvic examination reveals a normal-sized uterus, and bimanual examination reveals that the ovaries are not palpable. A rapid pregnancy test is negative. What Is the Most Likely Diagnosis? A. Premature ovarian failure B. Pituitary tumor C. Cushing's syndrome D. Polycystic ovarian syndrome E. Hypothyroidism

Correct answer: D The patient has secondary amenorrhea. Her irregular menstrual cycles interspersed with amenorrhea, obesity, and signs of hyperandrogenism are all consistent with polycystic ovarian syndrome (PCOS). The diagnosis requires 2 of the following criteria: ovulatory dysfunction, hyperandrogenism, and ultrasound evidence of polycystic ovaries. Laboratory abnormalities typically include LH:FSH > 2 and mild elevation in testosterone. Hypothyroidism also causes irregular menstruation, but this patient has no signs or symptoms of hypothyroidism such as cold intolerance, constipation, or coarseness of hair and nails. In addition, hyperandrogenism is not associated with hypothyroidism. Pituitary tumors often can lead to elevated prolactin levels, and alarm symptoms include headaches and vision changes; however, obesity and hyperandrogenism do not commonly occur. A prolactinoma is often caused by medications and can present with galactorrhea. Premature ovarian failure is often associated with estrogen deficiency, and patients usually have related symptoms including hot flashes, vaginal dryness, and decreased libido.

A 19-year-old man comes to your office with a complaint of a sore throat. His illness began 3 days ago with a sore throat, followed by persistent fevers of 102°F. He notes that he has a mild cough, some mild nausea, and no rhinorrhea. He reports odynophagia but no drooling, neck swelling, or difficulty breathing. He does not smoke or use drugs. He has not been sexually active for 6 months. His examination reveals a fever of 101.8°F, white exudates on his reddened tonsils, and no cervical lymphadenopathy. What Is the Most Likely Diagnosis? A. Group A streptococcal pharyngitis B. F necrophorum pharyngitis C. Laryngeal inflammation from gastroesophageal reflux disease D. Viral pharyngitis

Correct answer: D This young adult has symptoms of acute infectious pharyngitis, but he has no alarm symptoms. The next step is to assess the likelihood of GAS pharyngitis using the Centor Clinical Prediction Rule. He has 2 points (fever and tonsillar exudates). This Centor score has a likelihood ratio of 0.75. With a pretest probability for GAS infection in adults of 10% to 15%, the posttest probability is 8% to 12%. Fusobacterium necrophorum is another possible cause, which typically causes severe pharyngitis and is present in only 10% to 15% of young adults. Viral pharyngitis causes the majority of cases of acute pharyngitis. The acuity of the sore throat and fever make GERD an unlikely cause.

While taking a detailed history from a patient with "red eye," which of the following would not be considered an alarm symptom (ie, prompting concern for a potentially serious disease requiring referral to an ophthalmologist)? A. A gritty feeling in the eye B. Decrease in visual acuity C. Inability to keep the eye open D. Photophobia

The correct answer is A A gritty feeling is common in conjunctivitis and is not considered an alarm symptom.

Your patient, a 42-year-old woman, has a history of progressive dysphagia to solids and liquids for approximately 10 years. Recently, she has begun to lose weight and now sleeps sitting up to avoid regurgitating liquid. She notes chest fullness with some pain. What is the most likely diagnosis? A. Achalasia B. Peptic stricture C. Esophageal cancer D. Oropharyngeal dysphagia

The correct answer is A Achalasia is a motor (motility) disorder of the esophagus in which there is failure of the lower esophageal sphincter (LES) to relax along with abnormal movement of the esophagus in response to swallowing. The impairment of the LES to relax causes a functional obstruction of the esophagus, which is relieved when the pressure of the contents of the esophagus exceed the pressure of the sphincter or through occasional intermittent spontaneous relaxations of the LES. Achalasia is rare; it affects men and women equally. Dysphagia for both solids and liquids is the most common symptom. Other symptoms include regurgitation of food, chronic cough, chest pain, and weight loss. Peptic stricture is less likely given the duration of symptoms, lack of typical gastroesophageal reflux disease symptoms, and complaints of dysphagia to both liquids and solids, which typically occurs in patients with an esophageal motility disorder. Esophageal cancer is unlikely due to the duration of symptoms and the presence of dysphagia to both liquids and solids. This patient does not have symptoms suggestive of oropharyngeal dysphagia.

A 65-year-old diabetic man presents 1 week after hospitalization and surgery for an inflamed appendix. He was hospitalized for 3 days. He underwent intravascular catheter placement and received a urinary catheter and prophylaxis for deep venous thrombosis. He started taking antibiotics with surgery and quickly improved. His catheter was removed, and he was able to ambulate without pain. He was discharged home. He now presents to the emergency room with 3 days of fever, lower abdominal pain, and dysuria. He has no rash, leg pain, dizziness, chest pain, or shortness of breath. Which of the following is the most likely iatrogenic cause of his fever? A. Placement of urinary catheter B. Prophylaxis for deep venous thrombosis C. New medications D. Anesthesia for his surgery

The correct answer is A Acute fever after hospitalization raises concern for nosocomial infection, pulmonary embolism from immobilization, or drug fever due to a new medication. The dysuria and abdominal pain raise the concern of urinary tract infection, and this is most commonly associated with recent catheter placement. Asking a patient whether a catheter was placed can help narrow the search for cause of fever even in patients without dysuria. Always consider pulmonary embolism after a hospitalization. However, this patient received venous thromboembolism prophylaxis, which makes this less likely. He also denies chest pain or shortness of breath. Recent anesthesia raises the possibility of malignant hyperthermia; however, the time course of fever would be immediate, not delayed, and the peak of temperature is usually higher. Medications such as antibiotics can cause fever even within this short time frame. Drug fever should remain on the differential should the work-up for infection return negative. Evaluation should focus on urinary and blood cultures given that nosocomial urinary infections can quickly progress to sepsis.

A 65-year-old man with a prior myocardial infarction presents to your office with intermittent chest pain for the last 10 days. The pain is diffuse and precordial, increases with inspiration or lying down, and is relieved by sitting up and leaning forward. Sometimes the pain can last for hours at a time. It is somewhat different from his prior heart attack pain, but bothers him nonetheless and can bring on a sensation of shortness of breath when it is severe. Three weeks ago, he had what felt like the flu, but this resolved after a few days. The pain is not clearly related to exertion. What is the most likely diagnosis? A. Pericarditis B. Unstable angina C. Musculoskeletal chest pain D. Pulmonary embolism

The correct answer is A Although the patient has had a prior myocardial infarction, the features of his current chest pain (nonexertional, pleuritic, and positional) do not suggest myocardial ischemia. The recent flu-like illness and relief of the pain upon sitting up and leaning forward suggest pericarditis.

A 45-year-old woman has made an appointment to see you for an 8-week history of redness and thickening overlying the skin on her left breast. She stopped nursing 3 months ago. She initially received a course of antibiotics for mastitis, but the symptoms only partially improved. A diagnostic mammogram 4 weeks ago revealed dense breast tissue and skin thickening but no obvious mass. She has no other medical problems, and her family history is significant for a sister who was diagnosed with ovarian cancer at age 50 and a paternal aunt with breast cancer in her 60s. Which aspect of this patient's history is most consistent with a benign condition? A. Recent lactation B. Absence of mass on mammogram C. Partial response to antibiotics D. Family history of ovarian cancer

The correct answer is A In a patient with breast skin changes, a history of recent lactation may suggest mastitis. In this case, however, the worrisome features for breast cancer include the partial response to antibiotics and a paternal family history suggestive of hereditary breast and ovarian cancer. The absence of a mass on breast imaging is not reassuring and should not prevent further work-up in patients with persistent skin changes. Frequently, inflammatory breast cancer will cause diffuse skin changes and lack of a discrete mass on imaging.

A 66-year-old man with history of 2 prior myocardial infarctions, coronary artery bypass grafting, hypertension, and congestive heart failure suddenly turns pale and slumps over in his chair and onto the floor while eating breakfast with his wife. He appears cold, sweaty, clammy, and unconscious. She phones 911. By the time the emergency medical technicians arrive 15 minutes later, he is groggy but awake. His pulse and blood pressure are normal, and he refuses to go with them to the hospital because he feels fine. Which of the following most suggests that he has a serious condition? A. History of heart disease B. Loss of consciousness C. Sweating D. Pallor

The correct answer is A Loss of consciousness defines syncope. Sweating and pallor are common symptoms of vasovagal or neural reflex syncope. This patient has a history of significant structural heart disease, which is an alarm feature. The presence of heart disease is a strong independent risk factor for a cardiac cause of -syncope with sensitivity approaching 95% in some studies (which means the absence of heart disease may be helpful in excluding a cardiac cause)

Mr. Adams, a 66-year-old man, is a long-standing patient of yours. He has never described headache to you before. He schedules an urgent care visit with you to discuss the recent onset of recurring headaches. Which of the following features, if present, would be most concerning for a diagnosis of giant cell (temporal) arteritis? A. Scalp tenderness B. Vertex pain C. Unilateral pain D. Photophobia E. Lacrimation with headache

The correct answer is A Scalp tenderness and jaw claudication are the most specific features of giant cell arteritis. The pain can be anywhere in the head but is most commonly temporal, not vertex. Photophobia is characteristic of migraine; it may also occur in meningitis, although that would be an acute, rather than recurring, headache. Unilateral pain is nonspecific. It can occur with giant cell arteritis, but also with migraine and cluster headache. Lacrimation is a feature of cluster headache, not giant cell arteritis.

Your patient presents complaining of chronic scaling and erythema around his nose and eyebrows. Upon further questioning, he also reports dandruff of the scalp and a family history of atopy. What is the most likely diagnosis for his facial scaling? A. Seborrheic dermatitis B. Tinea faciei C. Psoriasis D. Atopic dermatitis

The correct answer is A Seborrheic dermatitis is very common. Many patients will not report it as a concern, but you may notice it during your examination. This eruption can present on the scalp, face, and chest. It is easy to treat but cannot be cured. Tinea facie is usually annular in appearance and can occur anywhere on face. The lesions of psoriasis are well-demarcated plaques that have a silver scale. Psoriasis can occur anywhere on the body; however, it favors the scalp, elbows, knees, and buttocks. Atopic dermatitis is always pruritic. If located on the face, then generally it presents as lichenified plaques/hyperlinearity around the eyes and erythematous cheeks.

A patient complains of left flank pain for several days. Which of the following historical features suggests herpes zoster as the cause? A. Constant burning and tingling pain B. Recent cardiac catheterization C. Pain is worse after eating D. Radiation to the groin E. Association with nausea and vomiting

The correct answer is A The flank pain associated with herpes zoster, or "shingles," can be severe and, in its earliest stages, may occur before the characteristic dermatomal rash develops. For this reason, it is occasionally mistaken for other causes of flank pain. A patient with herpes zoster may complain of burning pain and may describe a tingling or an electrical sensation. Flank pain that occurs after cardiac catheterization may indicate retroperitoneal hemorrhage and should be evaluated immediately. Pain radiating to the groin and nausea are suggestive of nephrolithiasis or pyelonephritis.

A 67-year-old man with a 40 pack-year smoking history complains of shortness of breath for the past 6 months and swelling in his legs. He often wakes up at night gasping for breath and feels most comfortable sleeping on 3 pillows. He has no past history of COPD or CHF, but he has diabetes and hypertension. What is the most likely diagnosis? A. Congestive heart failure B. Chronic obstructive pulmonary disease C. Asthma D. Lung cancer E. Pulmonary embolism

The correct answer is A The patient has several coronary artery disease risk factors (male sex, age > 55 years, smoking history, diabetes, and hypertension) and describes paroxysmal nocturnal dyspnea, 3-pillow orthopnea, and lower extremity edema, the classic symptoms of congestive heart failure (CHF). The patient's significant smoking history also raises suspicion for chronic obstructive pulmonary disease. Asthma is less likely given the patient's age of onset. The patient's smoking history puts him at increased risk of lung cancer. Pulmonary embolism can be difficult to diagnose. Although the other diagnoses cannot be definitively excluded, the clinical picture is most consistent with CHF.

An otherwise healthy 36-year-old woman comes to your office because of constipation for 2 years. Although she has a bowel movement every day, she has to strain for several minutes to pass stool. She admits with embarrassment that she occasionally has to insert her fingers in her vagina and push posteriorly in order to evacuate the stool. Her stools, when finally passed, are sometimes soft. She says over-the-counter laxatives "just don't seem to do much." When she has tried enemas in the past, they have "stayed inside" and have been difficult to evacuate. Which of the following questions would be most important to ask? A. Obstetric history B. Depression screen C. Medication list D. History of weight gain

The correct answer is A The patient's history suggests a defecatory disorder, such as pelvic floor dyssynergia, particularly because she reports sometimes having to manually evacuate even soft stool. An obstetric history is important to elicit because multiparity and rectovaginal trauma during delivery can disrupt the normal function of the pelvic floor. Specific questions to support the diagnosis of defecatory disorder include: "Do you feel like your bowels are blocked? Do you have difficulty letting go or relaxing your muscles to have a bowel movement?" The possibility of sexual abuse should also be explored. Depression-related constipation is unlikely to require digital manipulation to assist evacuation. A medication history is routine, but medications that cause constipation usually do so by slowing colonic transit, resulting in hard stools.

A 30-year-old man complains of recurrent, self-limited episodes of "bloody" urine and abdominal pain. He has a 10-year history of smoking one-half pack of cigarettes per day. His grandfather died of kidney disease, and his paternal aunt and father are currently treated on the "kidney machine." What is the most likely diagnosis? A. Polycystic kidney disease B. Bladder cancer C. Ureteral calculus D. IgA nephropathy E. Urinary tract infection

The correct answer is A This patient's family history is positive for kidney disease in his grandfather, father, and paternal aunt, which suggests a familial form of kidney disease. Autosomal dominant polycystic kidney disease (PCKD) is common, occurring in approximately 1 in 500 births. Patients with PCKD can present with recurrent gross hematuria and abdominal pain of one of the multiple cysts found in the grossly enlarged, often football-sized kidneys. Although smoking is an alarm feature for bladder cancer, this disease rarely occurs in patients less than 40 years old. A ureteral calculus would be expected to cause intense flank or abdominal pain, and unlike PCKD, there is not a clear genetic predisposition. Likewise, there is no clear genetic cause for IgA nephropathy. The self-limited nature of the patient's symptoms (without treatment) does not support the diagnosis of urinary tract infection.

Which of the following is not a cause of syncope? A. Carotid distribution transient ischemic attack B. Severe aortic stenosis C. Pulmonary embolism D. Paroxysmal supraventricular tachycardia E. Left atrial myxoma

The correct answer is A Transient ischemic attacks (TIAs) are transient and, like syncope, self-limited. However, carotid artery distribution TIAs typically last longer and are associated with localizing neurologic signs and symptoms. Vertebrobasilar TIAs may cause loss of consciousness but are accompanied by hemianopsia or other focal neurologic features such as vertigo, dysarthria, and diplopia. In general, cerebrovascular disease is an unusual cause of syncope and should not be considered unless there are focal neurologic symptoms or signs. Only subclavian steal causes loss of consciousness compatible with syncope, often triggered by arm exercise. All other options are known causes of syncope. Severe aortic stenosis causes exercise-induced syncope via decreased cerebral perfusion due to obstruction of left ventricular outflow and cardiac output in the face of peripheral vascular dilatation in exercising muscles. Pulmonary embolism raises pulmonary arterial pressures causing functional obstruction to right ventricular outflow, which leads to "underfilling" of the left ventricle and decreased cardiac output. Left atrial myxoma causes transient obstruction of inflow to the left ventricle and thus decreased cardiac output, leading to decreased cerebral perfusion that is often positional.

A 60-year-old woman presents with 2 months of progressive, crampy, lower abdominal pain and cyclical diarrhea. Which of the following features, if present, would be most concerning for endometrial cancer? A. Fever or chills B. Abnormal vaginal bleeding C. Dyspareunia D. Multiple sexual contacts

The correct answer is B Abnormal vaginal bleeding in a postmenopausal woman is worrisome for endometrial cancer. Fever and chills in patients with chronic pelvic pain suggest an infectious or inflammatory etiology rather than malignancy. Dyspareunia and sexual promiscuity are not associated with endometrial cancer.

A 60-year-old man sees you with concerns about hearing loss. On further questioning, he cannot remember an acute onset of hearing loss and believes it has been gradual over time. Which of the following features would make you the most concerned for acoustic neuroma? A. Fever B. Unilateral hearing loss C. Rapidly progressive hearing loss D. Fluctuating hearing loss E. Itchy ears

The correct answer is B Acoustic neuroma causes a gradual onset unilateral or asymmetric hearing loss. Fever is typically associated with otitis media or meningitis. Itchy ears raise the possibility of psoriasis. Fluctuating hearing loss can be seen in a variety of conditions but not typically with acoustic neuroma.

The captain of the high school cheerleading team presents to clinic with a chief complaint of several months of shortness of breath upon exertion. She is unable to identify any triggering symptoms. She denies hormonal contraceptive use. Which of the following is the most important aspect of the history to elicit? A. Recent bee sting B. Menorrhagia C. Recent immobilization D. Exposure to asbestos

The correct answer is B After cardiac and pulmonary causes, anemia is the most common cause of shortness in breath. In young women, menorrhagia is a common cause of anemia and should be considered in this case. The prolonged nature of the patient's symptoms is inconsistent with anaphylaxis. Chronic pulmonary embolism is unlikely in this otherwise healthy teenager. Although exposure to asbestos can cause infiltrative lung disease, pleural disease, or malignancy, it typically does not develop for decades.

A 65-year-old postmenopausal woman notes a 1-month history of a painless right breast mass. She denies nipple or skin changes. She has a history of hypertension, arthritis, and fibrocystic breast disease. She had a right breast biopsy 10 years ago for a breast lump, which revealed atypical ductal hyperplasia but no invasive or in situ cancer. She is on an antihypertensive, acetaminophen, and hormone replacement therapy (HRT). Her family history is significant for her mother, who had breast cancer in her 70s. Which of the following historical features does not increase the risk of breast cancer? A. Age B. History of fibrocystic disease C. Prior breast biopsy D. HRT use E. Breast cancer history in mother

The correct answer is B Breast cancer risk factors in this vignette include increasing age; history of a prior breast biopsy, particularly with atypia; exogenous estrogen use (HRT) in postmenopausal woman; and family history, especially in a first-degree relative. A history of fibrocystic disease does not increase a woman's risk of developing breast cancer. However, most women diagnosed with breast cancer do not have these risk factors, with the exception of increasing age. Therefore, a patient who presents with a discrete breast mass needs further work-up regardless of the findings on imaging (see Figure 47-1).

A 35-year-old woman with a negative pregnancy test is concerned about her menstrual bleeding. Which of the following symptoms associated with the bleeding is not suggestive that her bleeding is ovulatory in nature? A. Bloating B. Cold intolerance C. Breast tenderness D. Moodiness

The correct answer is B Cold intolerance suggests hypothyroidism, which, when it causes menstrual irregularities, causes increased bleeding. The other symptoms are all hormonally related to the menstrual cycle and, when present with bleeding, strongly suggest that the bleeding is ovulatory in nature.

You see a 22-year-old man with recent onset of solid food dysphagia. His symptoms have gradually worsened, and he has experienced several bouts of food impaction. He has a history of asthma and allergic rhinitis and believes that he has multiple food allergies as well. What is the most likely cause of his dysphagia? A. Achalasia B. Eosinophilic esophagitis C. Schatzki ring D. Esophageal spasm

The correct answer is B Eosinophilic esophagitis (or allergic esophagitis) is an increasingly common disorder of the esophagus that is characterized by eosinophilic infiltration of the esophagus due to allergic or idiopathic causes. Children and young adults are most commonly affected, although it can occur at any age. Patients most commonly report dysphagia that is frequently complicated by food impaction. A history of atopy (eg, rhinoconjunctivitis, asthma, dermatitis) is commonly present. Strictures frequently occur and can be present throughout the esophagus. Multiple mucosa rings are commonly present at endoscopy, and biopsies typically reveal greater than 20 eosinophils per high-power field. Schatzki ring could be a cause of this patient's symptoms. However, given his age, gender, recent onset, and history of atopy, the most likely diagnosis is eosinophilic esophagitis. Achalasia and esophageal spasm are unlikely to explain his symptoms. These motility disorders rarely present with food impaction, and patients generally have dysphagia to both solids and liquids.

A 67-year-old woman comes to your office with sudden onset of unilateral headache and decreased visual acuity. The symptoms started the previous evening while she was watching television in a dark room. She describes the headache as a deep throbbing pain behind her right eye. She took ibuprofen and a sleeping pill and went to sleep but awakened early this morning with more pain and noted that her eye was diffusely red. In the waiting room, she developed nausea and had 1 episode of emesis. She describes blurred vision and seeing halos around lights. She denies significant discharge other than tearing of the affected eye. She denies a foreign body sensation, photophobia, or fever. Of the following, which diagnosis is most consistent with this patient's presentation? A. Subconjunctival hemorrhage B. Acute angle-closure glaucoma C. Bacterial conjunctivitis D. Scleritis

The correct answer is B Haloes around lights, decreased visual acuity, headache, and onset in low-light situations (where mydriasis can result in further blocking of the narrow angle, preventing the outflow of aqueous humor) are classic symptoms of acute angle-closure glaucoma. Although scleritis can diminish visual acuity and cause deep pain, it should not result in nausea and vomiting or the appearance of halos around lights.

A 56-year-old man sees you for a long-standing cough. He has had a cough for at least 6 months. On a review of systems, he denies wheezing, nasal congestion, weight loss, fevers, or hemoptysis. He has occasional heartburn characterized by a midline retrosternal burning, especially after heavy meals or chocolate. He has had frequent hoarseness over the past month. Another physician did a preliminary work-up that included a chest x-ray and blood and urine studies. All of these tests were normal. What should you do next? A. Do additional questioning/testing for possible bronchial asthma B. Do additional questioning/testing for possible GERD C. Do additional questioning/testing for possible infectious causes of cough D. Follow his symptoms and schedule a follow-up in 1 month

The correct answer is B His symptoms suggest GERD. Heartburn characterized by a midline retrosternal burning is a characteristic feature of GERD. Hoarseness may be a sign of severe disease. The patient should be treated empirically for this diagnosis. Depending on his response to therapy, additional testing might be required to confirm the diagnosis. Although asthma remains a possibility, he had no wheezing or other features to suggest this diagnosis. Based on his symptoms, the likelihood of an infectious cause of his cough is low.

You see a 40-year-old woman with throbbing headaches. She reports that the symptoms began approximately 2 months ago and have gradually worsened. They are always on the right side. On one occasion, she had a prolonged visual aura of zigzag lines that lasted for one day and persisted after the headache had resolved. Which of the following features is not an alarm symptom that should prompt concern for a serious cause of headache? A. New onset of headache at age 40 B. Throbbing pain C. Prolonged visual aura D. Worsening pain over 2 months E. Pain always on the right side

The correct answer is B Migraine headaches usually begin for the first time in adolescence. Migraine-like headaches that begin for the first time at age 40 raise concern about an arteriovenous malformation (AVM). Although migraines are commonly unilateral, they should vary from one side to the other over a patient's lifetime. Pain always on the same side also suggests the possibility of AVM, although in some patients, this will prove to be an atypical manifestation of ordinary migraine. Pain that worsens over 2 to 3 months is worrisome for a brain tumor. Finally, visual aura should not last for more than 1 hour, so a prolonged aura raises concern for AVM as well. Throbbing pain is a cardinal feature of migraine and by itself is not an alarm symptom.

A 5-year-old boy is brought in to the office by his mother because of 2 days of right ear pain. The mother reports that he has been crying and pulling on the ear. He has a temperature of 39.7°C. What is the most likely diagnosis? A. Otitis externa B. Otitis media C. Foreign body in the ear D. Malignant otitis externa

The correct answer is B Otitis media is the most common cause of ear pain in young children. Typical symptoms include pulling or rubbing the affected ear and crying or irritability. Fever is usually, but not always, present. A foreign body can cause ear pain in a young child but would not typically cause fever. Otitis externa is usually more modest pain, often associated with a discharge from the ear and with exposure to water ("swimmer's ear"). Malignant otitis media is a rare disease, usually seen in older patients with diabetes or a compromised immune system.

You see a 45-year-old male construction worker for a periodic health examination. On review of symptoms, he reports subjective hearing loss. Concerned about noise-induced hearing loss, you assess for modifying symptoms. Which sounds or situation would you expect the patient to be LEAST likely to have difficulty with? A. High-pitched sounds B. Low-pitched sounds C. With background noise D. TV or radio E. A conversation in a group of people rather than one-on-one

The correct answer is B Patients with noise-induced hearing loss may report difficulty hearing high-pitched sounds, the TV, or radio. There will also be difficulty hearing when there is background noise or in a conversation with a group of people. Difficulty hearing low-pitched sounds is more typical of Ménière's disease or migraine as the etiology of hearing loss.

A 48-year-old man who has been your patient for 14 years and is in good general health reports continued episodes of rectal pain that last a few seconds and then disappear completely. The pain seems to be worse with sitting than with standing or lying down. Which of the following symptoms suggests a diagnosis of proctalgia fugax? A. Anorectal pain associated with dysuria and urinary urgency B. Sudden, brief paroxysms of pain C. Bright red blood per rectum D. Intermittent anal discharge

The correct answer is B Proctalgia fugax is characterized by sudden, severe pain in the anorectal region that typically lasts seconds to minutes and then disappears completely. It does not have a well-understood etiology and is a diagnosis of exclusion. In patients with proctalgia fugax, there are no organic explanations for the pain and no other signs or symptoms. It may correlate with stressful life events and anxiety, and the prevalence may be higher in patients with a perfectionistic personality type. Consider the diagnosis in patients with sudden, severe pain that only lasts a short time and no anorectal pain between episodes. A confident diagnosis first requires exclusion of other causes of the pain.

A 67-year-old hospitalized man complains of shortness of breath for the past hour. Although lying in the bed, he is breathing quickly and appears nervous. He was admitted 4 days earlier after being hit by a car and was found to have a displaced tibial fracture, which required immediate operative intervention. Chest radiograph at admission showed a 5-cm lung mass suspicious for cancer. The patient has a 60 pack-year smoking history. He has had a persistent nonproductive cough for the past several months but denies any other symptoms. What is the most likely cause of his dyspnea? A. Lung mass B. Pulmonary embolism C. Rib fracture D. Anxiety E. Deconditioning

The correct answer is B Pulmonary embolism (PE) is the most likely cause of dyspnea in this patient. The significant smoking history and suspicious mass on chest radiograph are very concerning for lung cancer. The patient's acute dyspnea is most likely due to a venous thromboembolism (deep venous thrombosis) originating in the leg. The modified Wells criteria are clinical prediction rules that can be used to classify the patient as likely (score > 4) or unlikely (score ≤ 4) to have a PE (see van Belle et al reference in Suggested Reading section of Chapter 25). This patient has a Wells score of 4.5, placing him in the category of likely to have a PE because he has had recent surgery (1.5 points) and does not have an alternative diagnosis that is more likely than PE to explain his symptoms (3 points). If the patient also has lung cancer, leading to a hypercoagulable state, this would further increase his Wells score (1 point). Smoking has also been associated with an increased risk of PE. Rib fractures are generally highly painful and lead to shallow, hesitant respirations and, if displaced, can cause pneumothorax. In this case, rib fracture is unlikely given the delay between the patient's accident and the onset of dyspnea. Anxiety should not be diagnosed until other causes have been ruled out. Deconditioning will be a concern for this patient during his recovery but is unlikely to present acutely.

You are evaluating a 65-year-old man in clinic for sudden onset of severe flank pain. Which of the following features is most alarming and should prompt an urgent evaluation? A. History of kidney stones B. Patient is taking warfarin (for stroke prevention due to atrial fibrillation) C. Radiation to the groin D. Pain started after he picked up a heavy box E. Hematuria

The correct answer is B Sudden onset of severe flank pain in a patient with atrial fibrillation suggests the possibility of a renal embolism or infarction. The patient is also taking warfarin, which increases suspicion for retroperitoneal hemorrhage. Pain radiating to the groin is typical of nephrolithiasis but can also be seen in pyelonephritis. Pain that started after picking up a heavy box suggests musculoskeletal injury. Hematuria can be associated with both pyelonephritis and nephrolithiasis.

A 30-year-old man comes to the emergency room complaining of mild dyspnea and diffuse chest pain that started suddenly 3 hours ago and worsens with inspiration. While sitting in the waiting room, he has an episode of prolonged coughing that produces small amounts of bright red blood. The physical examination reveals tachycardia and a testicular mass. He has no other medical problems and takes no medications. He just returned from a 12-hour drive to visit his family. What is the most likely diagnosis? A. Bronchitis B. Pulmonary embolism C. Pneumonia D. Tuberculosis

The correct answer is B The most likely diagnosis is pulmonary embolism, which typically presents with acute chest pain and dyspnea. Prolonged immobility and hypercoagulability are risk factors for pulmonary embolism. The patient was immobile for a prolonged period of time during his recent travel. Furthermore, his testicular mass most likely represents a malignancy, which increases his risk for hypercoagulability. Bronchitis and pneumonia are less likely as he did not complain of fevers or increased sputum production. He does not have risk factors for tuberculosis

A 19-year-old woman with a history of irregular menses since menarche presents to your clinic accompanied by her mother for evaluation of absent menses for the past 9 months. She has never had an evaluation of her abnormal menses. She is on the gymnastics team at her university, and her body mass index is 17. On physical examination, she has no hirsutism or acne. A serum pregnancy test is negative. What is the most likely diagnosis? A. Polycystic ovarian syndrome B. Exercise-induced hypothalamic dysfunction C. Pregnancy D. Delayed puberty E. Nothing is wrong

The correct answer is B The patient has a secondary amenorrhea. An athletic woman with amenorrhea and low body mass index must raise suspicion for exercise-induced hypothalamic dysfunction. Inadequate caloric intake to match energy expenditure can lead to decreased gonadotropin-releasing hormone secretion causing secondary amenorrhea. Consideration of an underlying eating disorder would also be important. She has no signs or symptoms of hyperandrogenism that would suggest polycystic ovarian syndrome. Pregnancy was ruled out as the cause of secondary amenorrhea. Delayed puberty is a type of primary amenorrhea. Finally, it is not "normal" to have secondary amenorrhea.

A 47-year-old construction worker reports intermittent dizziness, occurring twice weekly for 15 to 60 minutes, accompanied by severe, nonpulsatile tinnitus for the past year. He recently lost his job because he injured his back after falling off a ladder. His disability application was denied on the first application because the evaluator was concerned that he was "faking" the severity of his injury. He is upset that he cannot provide for his family but has begun to restrict his driving and is afraid to climb buildings. His wife is threatening to leave him, because he is a changed man. In the office, he denies dizziness or tinnitus. The physical examination and laboratory tests are normal. What is the most likely diagnosis? A. Otosclerosis B. Ménière's disease C. Labyrinthitis D. Intracranial arteriovenous malformation (AVM)

The correct answer is B This patient has classic presentation of Ménière's disease, described by French physician Prospere Ménière in the 1860s. This disorder is characterized by episodic vertigo, nonpulsatile tinnitus, and progressive fluctuating sensorineural hearing loss (often to low frequencies). The hearing loss and tinnitus may be unilateral (more commonly) or bilateral. During episodes, the patient typically has nystagmus. Symptoms can be mild to incapacitating. The cause of Ménière's disease is unknown but may be related to endolymphatic hydrops, an excess of fluid and pressure in the middle ear. There is an association with herpes simplex infection and with migraine headaches. Labyrinthitis may also present with episodic tinnitus, vertigo, and hearing loss but typically follows a viral infection and is self-limited. Otosclerosis causes bilateral progressive conductive hearing loss and tinnitus. Arteriovenous malformation or other vascular causes would be associated with pulsatile tinnitus.

A 65-year-old man comes to see you for dizziness. He states it occurs upon standing. He describes the feeling of "almost passing out," but this is averted by seating himself. He did have diarrhea for a few days after taking care of his sick grandchildren over the weekend. He continues to take his blood pressure and cholesterol medications. Which category of dizziness is most consistent with this patient's symptoms? A. Vertigo B. Presyncope C. Light-headedness D. Disequilibrium

The correct answer is B This patient has presyncope due to volume depletion and concomitant use of antihypertensive medications. The clinical history is consistent with brief periods of diminished cerebral perfusion from orthostatic hypotension.

In one of your acute care openings, you see a 72-year-old man with hypertension, diabetes mellitus, and chronic tobacco use. He awakened without any problems but developed the acute onset of dizziness during breakfast. He can barely walk. He notes double vision. What would be the most reasonable next step in caring for this patient? A. Return home and monitor symptoms of vestibular neuritis B. Send him to the emergency department for urgent evaluation of acute vertebrobasilar infarct C. Perform maneuvers in the office to realign inner ear otoliths for BPPV D. Hold all blood pressure medications to prevent syncope

The correct answer is B This patient is having an acute vertebrobasilar infarct and has numerous red flags. He has multiple stroke risk factors, including diabetes mellitus, hypertension, and tobacco use. The sudden onset of dizziness, severe ataxia, and double vision are all alarm symptoms. He requires an urgent evaluation for a serious neurologic cause.

Mrs. Jones, a 23-year-old G1P0 woman, sees you at 30 weeks of gestation because of severe pain with defecation. Pain occurs with bowel movements, and she has noted blood on the toilet paper. Which of the following is the most helpful when distinguishing between an anal fissure and a thrombosed external hemorrhoid? A. The color of the blood B. A tender lump C. Pain that wakes the patient up at night D. Constitutional symptoms

The correct answer is B This patient is suffering from an anal fissure. The typical presentation includes severe pain during and shortly after defecation, blood-streaked stool or blood on the toilet paper, and secondary constipation. It is caused by a tear in the anal canal, most commonly located in the posterior midline. However, anal fissures associated with pregnancy are most often in the anterior midline. The split in the dermal lining occurs distal to the dentate line where sensory nerve fibers run and is therefore very painful. A thrombosed external hemorrhoid can also cause severe pain and bright red blood per rectum; however, a distinguishing characteristic would be the presence of a tender, swollen lump.

You are seeing an 80-year-old woman in clinic with intermittent fever for 5 weeks. Her fevers have been as high as 102.9°F and respond to antipyretics. She has noted proximal muscle aches as well as some joint stiffness of her hands. Two weeks ago, she developed a left unilateral temporal headache and noted pain in her jaw after chewing for a few minutes. She now has loss of vision in her left eye. She has not been hospitalized recently and has not travelled outside of her state. She has no sick contacts and does not use intravenous drugs. What diagnosis do her clinical features suggest? A. Pulmonary embolism B. Giant cell arteritis (GCA) C. Hodgkin's lymphoma D. Bacterial endocarditis

The correct answer is B This patient's long duration of fever and arthritic symptoms suggest an inflammatory disorder. Her lack of sick contacts, recent travel, and drug use argue against but do not rule out an infectious process such as bacterial endocarditis. Symptoms that suggest vasculitis include arthritis and jaw claudication, which is the most specific clinical feature of this disease. Up to 42% of patients with giant cell arteritis (GCA) present with fever, and although not typically high, up to 15% of patients develop fever over 39°C. Loss of vision is likely due to a cranial arteritis and is a feared complication of this disease. Prompt recognition and diagnosis of GCA is critical in order to prevent visual loss.

A 54-year-old woman with diabetes mellitus, chronic kidney disease, and heart failure was admitted for bloody diarrhea due to a resistant strain of Escherichia coli, which is sensitive only to gentamicin. During her hospitalization, she requires increasing doses of diuretics for fluid overload. Two days later, she reports that her low-level non-pulsatile bilateral tinnitus has worsened to the point that it obscures conversation. Which of the following is unlikely to be implicated in exacerbating her tinnitus? A. Furosemide (diuretic) B. Gentamicin (for E coli infection) C. Acetaminophen (for pain) D. Aspirin (for secondary myocardial infarction prevention) E. Ibuprofen (for osteoarthritis pain)

The correct answer is C A variety of medications may cause nonpulsatile tinnitus, usually by damaging the cochlear apparatus of the inner ear. Among these medications are chemotherapeutic agents (especially platinum-based treatment), quinine, antibiotics (eg, gentamicin), diuretics (especially thiazides and high-dose furosemide), benzodiazepines, and NSAIDs or aspirin. Acetaminophen is unlikely to cause tinnitus. In this case, the most likely offending agents are the high-dose diuretics and gentamicin. The ototoxicity associated with these medications is usually reversible but occasionally can be permanent.

A 75-year-old man with a negative past medical history comes to your office complaining of a 3-day history of painless "bloody urine," which has now resolved. He is frightened because he has never experienced bloody urine. Which of the following would be considered an alarm symptom or feature in this patient? A. Deafness B. Burning upon urination C. Smoking history D. Recent travel to the Ukraine E. History of ureteral calculus

The correct answer is C Age greater than 40 to 50 years, male sex, and history of smoking are all alarm features for bladder cancer. Because gross hematuria itself can often cause mild to moderate dysuria, its presence alone is not an alarm symptom. Although deafness is associated with hereditary nephritis, this condition most often presents in pediatric or young adult patients and so would not be an alarm symptom in a 75-year-old man. Extended travel to or immigration from tropical Africa, the Middle East, Turkey, or India suggests the possibility of urinary Schistosoma haematobium infection or tuberculosis. These infections are not endemic in the Ukraine. This patient's hematuria was not accompanied by intense abdominal or groin pain, which is characteristic of patients with ureteral calculi.

A 19-year-old woman presents with one isolated episode of light spotting between her periods. Which of the following is not a likely cause of her bleeding? A. Cervicitis B. Traumatic intercourse C. Coagulopathy D. Oral contraceptive use

The correct answer is C Although a coagulopathy, especially von Willebrand disease, may be the cause of abnormal bleeding in young women, it would be uncommon for a single episode of light intermenstrual spotting to be the only clinical manifestation. If she had a coagulopathy, one would expect regular heavy bleeding and possibly bleeding from other sites. Any of the other diagnoses are possible in this patient and could cause a single episode of vaginal spotting.

Your 22-year-old patient has her fourth episode of yeast vaginitis in 3 months. You recommend that she be tested for which of the following? A. Hypothyroidism B. Trichomoniasis C. HIV D. Uterine fibroids

The correct answer is C Although candidal vaginitis can occur in completely normal women, recurrent infection should prompt investigation for underlying causes. Diabetes and HIV are particularly common causes. Recent antibiotic use can also cause candidal infection. Hypothyroidism and uterine fibroids are not associated with candidiasis, nor is trichomoniasis infection.

A 30-year-old mother of 2 healthy children presents with acute pelvic pain. Which of the following symptoms is most suggestive of a surgical emergency? A. Fever or chills B. Crampy abdominal pain C. Intense, progressive pain that started as repetitive transitory pain D. Deep dyspareunia

The correct answer is C An intense progressive pain that started as a repetitive transitory pain is a red flag for a surgical emergency like adnexal torsion. Fever and chills are alarm symptoms for serious medical causes like acute pelvic inflammatory disease and urinary tract infection but do not necessarily suggest surgical emergencies. Crampy abdominal pain and deep dyspareunia are typical features of dysmenorrhea and endometriosis, respectively, but they are not red flags for surgical emergencies.

A 20-year-old woman has just been diagnosed with bacterial vaginosis. She wants to know if this is a sexually transmitted infection. What do you tell her? A. All forms of vaginitis are sexually transmitted. B. Bacterial vaginosis implies that her partner has been intimate with someone else. C. Bacterial vaginosis occurs in women who have had sex, but it is not considered to be a sexually transmitted infection.

The correct answer is C Bacterial vaginosis is not thought to be a sexually transmitted disease but is associated with penile, digital, or sex toy insertion. Trichomoniasis, chlamydia, and gonorrhea are all sexually transmitted and require treatment of the partner as well. These infections imply that one partner has had intercourse with an infected party, although trichomonal infection can remain dormant for many years and does not imply recent other relationships. Consider partner treatment only if a woman has recurrent bacterial vaginosis. Candida is not considered sexually transmitted, although males with candidal balanitis can occasionally transmit infection, and Candida can be present under an intact foreskin.

A 38-year-old premenopausal woman sees you for blood-tinged bilateral nipple discharge for 2 weeks. The discharge has to be expressed from her breasts, does not appear spontaneously, and appears from several ducts. She is concerned because she stopped lactating 6 months ago. Which is the most worrisome feature of this patient's history? A. Age B. Recent lactation C. Bloody discharge D. Bilateral nipple discharge E. Nonspontaneous discharge F. Discharge from multiple ducts

The correct answer is C Bloody nipple discharge is more worrisome for breast cancer than milky, green, black, or brown discharge. The latter is commonly physiologic discharge or due to benign causes. Nipple discharge may continue for months after nursing has ceased. Other alarm symptoms include an underlying breast mass, unilateral breast involvement, discharge from one duct, and spontaneous discharge. The most common cause of bloody nipple discharge is a papilloma; less commonly, the cause may be in situ or invasive breast cancer.

A 60-year-old male smoker was brought to the ED after passing out while standing up. He regained consciousness a few minutes later but offered complaints of chest pain and dyspnea. The chest pain is right-sided and sharp and worsens with inspiration and cough. His cough is mildly productive and at times blood-tinged. On further questioning, he had felt generally well prior to this episode, although he had knee surgery 6 weeks ago. What is the most important diagnostic consideration? A. Pericarditis B. Pneumonia C. Pulmonary embolism D. Myocardial infarction E. Pneumothorax

The correct answer is C Chest pain and dyspnea with syncope should raise consideration for conditions that disrupt global cardiac output including acute myocardial infarction, other serious cardiovascular causes (eg, arrhythmia, valvular heart disease), and large pulmonary embolism that obstructs right ventricular flow. Here, the patient has had recent lower extremity surgery that increases the likelihood of deep venous thrombosis that can lead to pulmonary embolism. The pleuritic nature of chest pain and associated hemoptysis also suggest pulmonary embolism. Pleuritic chest pain can also be a feature of pericarditis, pneumonia, and pneumothorax, although the presentation usually does not include acute chest pain with syncope. The overall constellation points most strongly to pulmonary embolism.

A 25-year-old graduate student complains of a burning sensation in her throat and upper abdomen. She drinks 3 to 4 cups of coffee per day and often skips meals. Symptoms are often worse at bedtime. Which of the following symptoms would be most consistent with a diagnosis of gastroesophageal reflux? A. Weight loss B. Flatulence C. Chronic cough D. Vomiting

The correct answer is C Gastroesophageal reflux is a common condition that is often precipitated by ingestion of spicy foods or alcohol and classically causes a burning sensation in the epigastrium and radiating into the chest. Symptoms are worse upon lying down, and patients often complain of a bitter taste in their mouth. Chronic cough is a common complaint due to reflux of gastric contents. Nausea may occur, but vomiting is rare. Weight loss and flatulence are not features of GERD.

A 19-year-old man comes to your office with a complaint of a sore throat. His illness began 3 days ago with a sore throat, followed by persistent fevers of 102°F. He notes that he has a mild cough, some mild nausea, and no rhinorrhea. He reports odynophagia but no drooling, neck swelling, or difficulty breathing. He does not smoke or use drugs. He has not been sexually active for 6 months. His examination reveals a fever of 101.8°F, white exudates on his reddened tonsils, and no cervical lymphadenopathy. What is the next step in his evaluation and treatment? A. Symptomatic treatment (only) for presumed viral infection B. Antibiotics to cover group A Streptococcus and F necrophorum C. Perform rapid strep testing in the office and treat if positive D. Perform throat culture, start antibiotics, and then stop antibiotics if the culture comes back negative

The correct answer is C Given his low risk of GAS based on his Centor score, empiric antibiotics are not recommended. Similarly, treatment for possible F necrophorum in patients with mild pharyngitis is not recommended. A throat culture will take 2 days to come back, so empirically treating with antibiotics (while awaiting the results) would likely be excessive and unnecessarily expose the patient to the risk of antibiotic side effects. Thus, the correct answer is to perform a rapid strep test and treat if positive.

You are seeing a 67-year-old man who reports mild dysuria for the past 3 months. Which of the following symptoms would be least concerning for bladder cancer? A. One episode of bloody urine last week B. An 18 pack-year smoking history C. Associated symptoms of difficulty initiating urination and a weak urinary stream D. Associated symptoms of urinary frequency and urgency

The correct answer is C Gross hematuria in an older man is concerning for bladder cancer. Patients with bladder cancer usually experience gross hematuria that is intermittent and painless. Bladder cancer can also cause other bladder storage symptoms, such as urinary frequency and urgency. Although bladder cancer can rarely cause obstructive voiding symptoms, such as urinary hesitancy and weak stream, these symptoms commonly occur in men with benign prostatic hyperplasia and do not necessarily raise suspicion for cancer. Smoking and exposure to aniline dyes are important environmental risk factors for the development of bladder cancer.

A 30-year-old woman with previously normal menses presents to your clinic concerned about not having a period for 4 months following a dilation and curettage to treat heavy bleeding after a miscarriage. She reports having little energy, decreased appetite, and disturbed sleep patterns since the miscarriage. Which of the following is the first step in making the diagnosis? A. Check FSH levels B. Discuss her stress level in more detail C. Obtain a serum pregnancy test D. Perform hysteroscopy E. Order thyroid function levels

The correct answer is C In a woman of reproductive age with amenorrhea, the first step is to rule out pregnancy. A pregnancy test should be obtained before pursuing other potential causes. FSH levels and thyroid function tests are reasonable laboratory tests once pregnancy is ruled out. Emotional stress can cause amenorrhea secondary to hypothalamic dysfunction. Asherman syndrome is also possible given the patient's recent history of instrumentation, so a hysteroscopy should be considered if pregnancy is ruled out.

A 15-year-old high school student complains of a loud whooshing noise that worsens when taking tests or running long distances. He recently lost 15 lbs after joining the track team. No one else around him can hear the noise, and his classmates think he is making this up. His mother reports he has been behaving erratically and is worried that he is taking drugs. He feels like the noise is impairing his concentration and that he has a "bucket on his head." He is also upset about losing athletic competitions. What is the most likely cause of these sounds? A. Auditory hallucinations B. Myoclonic stapedius muscle spasm C. Patulous eustachian tube D. Malingering

The correct answer is C In patulous eustachian tube, the eustachian tube is either permanently open or opens intermittently. Breathing and other physiologic sounds are transmitted unimpeded to the tympanic membrane, often resulting in a "whooshing" sound. In some cases, the patulous eustachian tube is exacerbated by weight loss, as the fatty tissue that helps keep the eustachian tube open recedes. Similarly, decongestants can make this condition worse by drying up the secretions that may hold a partially patent eustachian tube closed. Key questions to ask would be if the "whooshing" sound occurs in time with his respiration and if sounds are muffled (as might be the case with allergies and sinus congestion). The clinician may also perform evocative maneuvers (eg, deep breathing exercises) in the office. These sounds are often amplified in quiet rooms and with vigorous exercise. Myoclonic stapedius muscle spasm usually presents with a clicking sound, up to 150 to 300 spasms per minute. Auditory hallucinations, migraine aura, and malingering are all diagnoses of exclusion.

In a patient with severe right-sided flank pain, which of the following lowers your suspicion for nephrolithiasis? A. Stabbing, severe pain B. Intermittent nature of pain C. Onset approximately 2 months ago D. Hematuria E. History of nephrolithiasis

The correct answer is C It is unlikely that this patient would tolerate the severe pain of nephrolithiasis for 2 months before seeking medical attention. The pain of pyelonephritis is often described by patients as intense spasm-like pain in the flank. Nephrolithiasis can be associated with gross hematuria. Finally, nephrolithiasis can recur, and a history of nephrolithiasis may prompt suspicion that another kidney stone has formed

Each of the following women presents with dysuria and has not had a medical evaluation for over a year. Which patient does not need testing for chlamydia? A. 19-year-old sexually active woman who consistently uses condoms B. 32-year-old woman with a new male sexual partner C. 26-year-old woman who has not been sexually active for the past year D. 27-year-old married woman who had chlamydia at age 26

The correct answer is C Sexually active young women with dysuria are at relatively high risk for urethritis caused by chlamydia or other sexually transmitted infections. In addition to urinalysis, such women should be queried about their risk for sexually transmitted infections. Sexually active women who are less than 25 years old, have a new male sexual partner, or report a prior sexually transmitted infection (STI) are at high risk and should undergo pelvic examination and chlamydia testing. Because condom use reduces, but does not eliminate, the risk of STIs, women who are at high risk should undergo chlamydia screening regardless of their use of barrier contraceptive methods.

A 15-year-old girl presents to your clinic for a routine annual physical. She expresses concern because she has not experienced menarche yet, but all of her girlfriends have. On examination, she has a body mass index of 21, normal pubic hair growth, and normal breast development. What is the most likely diagnosis? A. Primary amenorrhea B. Secondary amenorrhea C. Normal development D. Premature ovarian failure E. Polycystic ovarian syndrome

The correct answer is C She likely has normal development as she has not yet turned 16 and manifests normal secondary sexual characteristics (pubic hair and breast development). Primary amenorrhea is the absence of menarche by age 16 in the presence of otherwise normal secondary sexual characteristics. If she did not have evidence of secondary sexual characteristics, because she is older than 14, this would be considered primary amenorrhea. She does not have secondary amenorrhea because she has never had menses. Premature ovarian failure and polycystic ovarian syndrome are types of secondary amenorrhea.

A 16-year-old girl presents to you with painful blisters on her skin and oral and genital mucosa. Her eyes feel very dry and itchy. She has a temperature of 99°F and complains of malaise. One week ago, you prescribed trimethoprim-sulfamethoxazole for a urinary tract infection. On examination, she has large flaccid bullae, and her skin is painful to touch. What is your diagnosis? A. Bullous pemphigoid B. Scarlet fever C. Stevens-Johnson syndrome/toxic epidermal necrolysis D. Herpes simplex

The correct answer is C Stevens-Johnson syndrome/toxic epidermal necrolysis is typically a consequence of medication exposure. The most common offenders are antibiotics (particularly trimethoprim-sulfamethoxazole), nonsteroidal anti-inflammatory drugs (NSAIDs), anticonvulsants, and allopurinol. This is a life-threatening condition and must be diagnosed and treated. Patients with extensive skin sloughing often require attention in intensive care or burn units. Bullous pemphigoid is an autoimmune blistering disease. Typical lesions are tense bullae on an erythematous base. The mucosa is rarely involved. The uninvolved skin is usually not tender to touch. In scarlet fever, the skin is diffusely erythematous with a "sandpaper" feel. There are no bullae or vesicles. Other features that may be present are petechiae on the hard/soft palate and white exudate with prominent papillae on the tongue ("strawberry tongue").

Ms. Valenti is a 33-year-old patient of yours without significant medical history. She scheduled an urgent care visit to discuss hearing loss and a sensation of pressure in her ear that began 2 days ago. After further questioning, you are concerned for possible sudden sensorineural hearing loss. Which of the following symptoms is NOT usually associated with SSNHL? A. Tinnitus B. Vertigo C. Fever D. Unilateral hearing loss E. Rapidly progressive hearing loss

The correct answer is C Sudden sensorineural hearing loss is the sudden onset of unilateral rapidly progressive hearing loss that is often associated with tinnitus and vertigo. The presence of fever is atypical and would raise concern for otitis media or meningitis.

A 35-year-old male consultant tells you of several months of bilateral ear discomfort that is more pronounced on the left than the right. The pain is worse in the morning and worse when he chews. He has been working long hours recently to meet a deadline. What is the most likely diagnosis? A. Serous otitis B. Temporal arteritis C. Temporomandibular joint dysfunction D. Referred pain from tight neck muscles

The correct answer is C Temporomandibular joint dysfunction is a common cause of referred pain to the ear. It is typically chronic in its time course and aggravated by chewing. It can be unilateral or bilateral and is sometimes associated with stress. Although ear pain from serous otitis can be persistent, pain for several months would be unusual. Temporal arteritis occurs almost exclusively in patients over 50 years of age and is nearly always unilateral. Referred pain from tight neck muscles would generally be worse later in the day and is not likely to be worsened by chewing.

28-year-old man who recently emigrated from Mexico visits your clinic with a history of 4 months of cough. He has lost 15 to 20 lbs unintentionally and has occasional fevers and night sweats. The cough is dry, and he has coughed up a moderate amount of blood on 3 occasions. He has been a smoker for 2 years. There is no history of wheezing, rhinitis, or reflux symptoms. He currently works as a security guard in a chemical plant. What are the alarm features in this patient? A. History of smoking B. Duration of the cough C. Hemoptysis and weight loss D. Type of job

The correct answer is C The alarm features in this case are hemoptysis, weight loss, and fever. Although serious causes of cough are not common, the alarm features require immediate action and often additional testing. The duration of the cough provides a guideline for the differential diagnosis, but it is not an alarm feature. Although smoking and his occupation provide additional information, they are not alarm features.

28-year-old man who recently emigrated from Mexico visits your clinic with a history of 4 months of cough. He has lost 15 to 20 lbs unintentionally and has occasional fevers and night sweats. The cough is dry, and he has coughed up a moderate amount of blood on 3 occasions. He has been a smoker for 2 years. There is no history of wheezing, rhinitis, or reflux symptoms. He currently works as a security guard in a chemical plant. For the patient, what is the most likely diagnosis? A. Lung cancer B. Acute bronchitis C. Tuberculosis D. Chronic obstructive pulmonary disease

The correct answer is C The most likely diagnosis is tuberculosis. Patients typically report chronic cough with/without hemoptysis, fevers, night sweats, and weight loss. Patients come from endemic areas or are immunosuppressed. Lung cancer can cause similar symptoms, but typically it affects an older population. Acute bronchitis is of short duration and does not cause chronic cough or weight loss. Although the patient is a current smoker, he does not have the characteristic features of COPD or emphysema.

Your patient, a 77-year-old gentleman with a smoking history of 100 pack-years, presents with several months of dull rectal pain and tenesmus. This has been associated with blood in the stool and recently with significant constipation. Which of the following additional symptoms is most concerning for rectal cancer? A. Sharp, stabbing pain with defecation B. Vesicles in the perianal region C. Weight loss D. Recurrent respiratory infections

The correct answer is C There are approximately 40,000 new cases of rectal cancer each year in the United States. The incidence increases with age, becoming more common after age 50. Certain inherited conditions predispose patients to colorectal cancer, and thus obtaining a thorough family history is important. In addition, a number of environmental factors, such as cigarette smoking and alcohol consumption, may increase a person's risk of developing rectal cancer. Patients with rectal cancer may develop blood in their stool, signs of anemia, tenesmus, constipation, small-diameter stools, and weight loss. Typically, if patients feel pain, it is poorly localized. The red flags for rectal cancer are weight loss, fatigue, fever, and anemia

A 21-year-old woman presents to your office 6 months after returning from Mexico. While on her 1-week spring break, she did not travel to rural areas but spent time in a market sampling fresh fruits. She felt well until 2 months ago, when she developed fevers to 101.0°F associated with sweating and pruritus. The fevers last for hours to days and then are quiescent for several days before recurring. She has been seen in a walk-in clinic for these symptoms and underwent a urine test and blood work with no diagnosis. She now comes to you for a second opinion. She has no cough, chest pain, joint aches, or rash. She does note anorexia and occasional loose stools. She has lost 25 lbs. She has never lived or worked in an institutionalized setting. Which clinical feature is most useful in narrowing the differential diagnosis? A. Weight loss B. Loose bowel movements C. Pattern of fever D. Eating fresh fruit

The correct answer is C This patient has fever of unknown origin—a fever that lasts 3 weeks or longer with temperatures exceeding 100.9°F with no clear diagnosis despite 1 week of clinical investigation. This fever pattern is consistent with Pel-Ebstein fevers. Sixteen percent of patients with Hodgkin's disease present with this cyclic but unpredictable pattern. She also has several other symptoms that suggest malignancy or chronic inflammation including weight loss and sweats. Interestingly, pruritus is a common feature of Hodgkin's lymphoma at some point in the course of the disease; diagnostic work-up should include an examination for lymphadenopathy and a chest x-ray. The loose bowel movements in the setting of travel and eating uncooked food raise the possibility of traveler's diarrhea. However, the time course is long even for parasite infections, and gastrointestinal symptoms are not her primary symptom. Weight loss itself is a nonspecific sign of inflammation or infection and thus does not help narrow the differential diagnosis.

A 67-year-old man comes to your office for constipation. He says, "I just don't get it, Doc. I've been as regular as a clock all my life, but for the last few weeks, I've been getting more and more bound up." He relates increasing abdominal pain, nausea, loss of appetite, and an 11-lbs weight loss. He has noted some blood on the toilet tissue and in his stools. Which of the following is the most likely diagnosis? A. Hypothyroidism B. Depression C. Colon cancer D. Diverticulitis E. Spinal cord process F. Stricture

The correct answer is C This patient has several alarm symptoms worrisome for colon cancer, including advanced age. Other possibilities include diverticulitis, stricture, hypothyroidism, depression, and less likely, a spinal cord process. Focused history should include past history of abdominal radiation leading to stricture; poor concentration, low mood, and disturbed sleep suggesting depression; leg weakness and back pain possibly indicating a spinal cord process; and fever, which is consistent with both colon cancer and diverticulitis. Further evaluation is absolutely essential.

A 47-year-old man with a history of hypertension presents with a complaint of a sore throat for 5 months. It is mild and occurs daily. He denies postnasal drip, allergy symptoms, weight loss, and shortness of breath, but notes some mild heartburn and occasional hoarse voice. He has never smoked and does not drink alcohol. His examination shows no abnormalities of the oropharynx and no lymphadenopathy. What would be your next step in his evaluation? A. Rapid strep test to test for GAS infection B. Magnetic resonance imaging (MRI) of the neck C. Ear, nose, and throat (ENT) evaluation D. Empiric therapy with antiallergy medications

The correct answer is C This patient presents with chronic sore throat, which makes infectious etiologies unlikely. Common noninfectious causes include postnasal drip from allergic rhinitis or chronic sinusitis, laryngeal irritation from gastroesophageal reflux disease (GERD), and head and neck malignancies. He does have symptoms of GERD but no symptoms of postnasal drip or alarm symptoms for cancer. Add this to the normal examination, and the most likely diagnosis is reflux disease. The next step is an evaluation by ENT to examine his larynx and look for signs of inflammation or malignancy.

A 30-year-old woman has recurrent episodes of "smoky" brownish urine and dysuria, usually preceded by a cough, nasal congestion, and feverishness. These episodes spontaneously resolve without treatment and then she feels well. She has smoked one-half pack of cigarettes per day for 5 years. What is the most likely diagnosis? A. Bladder cancer B. Urinary tract infection C. IgA nephropathy D. Renal infarct E. Renal calculus

The correct answer is C This patient's symptoms are most characteristic of IgA nephropathy, the most common cause of primary glomerular nephropathy. Characterized pathologically by deposition of IgA in the mesangium of the kidney, it is present in 3% to 9% of biopsies (although not all of these patients are symptomatic). Bladder cancer is extremely rare in female patients under the age of 40, even those with a smoking history. Symptoms associated with urinary tract infections would not be expected to spontaneously resolve without treatment, and renal infarct and ureteral calculus are both accompanied by extreme abdominal or flank pain, which are not present in this patient.

Your patient, a 22-year-old woman with a presumptive diagnosis of migraine headaches, comes to see you for a routine periodic health examination. Which of the following locations of head pain would be most consistent with the diagnosis of migraine? A. Occipital B. Frontal C. Temporal D. Periorbital E. Vertex

The correct answer is C While there is some variation of pain location among patients with migraine, most patients will report unilateral pain that is most intense in the temporal area. Occipital and vertex pain raise the possibility of a cervicogenic headache. Periorbital pain can occur with cluster headache and with trigeminal neuralgia. Frontal pain is nonspecific.

A 22-year-old college student who is on the swim team reports 1 week of left ear pain. She noted a slightly yellow, watery discharge from the ear on her pillow this morning, which prompted her to make an appointment. She has no fever and mentions that the pain is worse when she pushes on the tragus. What is the most likely diagnosis? A. Perforated eardrum from otitis media B. Temporomandibular joint dysfunction C. Otitis externa D. Infected upper third molar tooth

The correct answer is C presents with mild to moderate ear pain, which is aggravated by pushing or pulling on the ear. The patient often reports a discharge from the ear (otorrhea) and a recent history of swimming. A patient with a perforated ear drum from otitis media usually reports severe ear pain that suddenly lessened, often immediately followed by purulent discharge from the ear. The patient with a perforated eardrum may report decreased hearing if the perforation is large. The pain of temporomandibular joint (TMJ) dysfunction is generally more chronic and is worsened by chewing. Patients with TMJ dysfunction may report a clicking sensation in the TMJ area, although many asymptomatic individuals note this as well. A patient with an infected molar tooth will report pain with chewing and may report increased pain with exposure to hot or cold foods or liquids.

A 10-year-old boy presents with very itchy blisters on his arms and legs. In addition, he woke up this morning with swelling and pruritus around his left eye. He is afebrile. On examination, he has several vesicles in a linear array on his arms and legs as well as redness and swelling of his left upper and lower eyelid. What is your diagnosis? A. Herpes zoster B. Herpes simplex C. Drug eruption D. Allergic contact dermatitis

The correct answer is D Allergic contact dermatitis classically occurs in the distribution described in the question and is most commonly due to poison ivy or a similar plant exposure. A linear distribution of lesions is an important clue. Careful patient history taking often reveals recent outdoor hiking, gardening, or playing. A history of physical exposure to plants usually precedes the eruption by 1 to 2 days. Herpes zoster is usually painful and follows a dermatomal distribution. Herpes simplex classically presents as painful vesicles clustered on an erythematous base. Drug eruption is generally more diffuse, not extremely pruritic, and not linear.

A 55-year-old woman is in your office with dyspareunia and vaginal discharge. She is widowed and just recently started a new relationship with a 60-year-old man. A possible cause of her symptoms includes which of the following? A. Atrophic vaginitis B. Trichomoniasis C. Candida vaginitis D. All of the above

The correct answer is D Atrophic vaginitis is particularly common in postmenopausal women, but other forms of vaginitis, including Candida vaginitis, can also occur after menopause. Be sure to obtain a sexual history; do not assume that older women are celibate.

A 56-year-old woman presents with vaginal bleeding and intermittent abdominal cramping. She does not track her menstrual cycle but states that her periods are very irregular. She thinks she last bled more than 9 months ago. Which of the following diagnoses should you exclude first? A. Menopause B. Dysfunctional uterine bleeding C. Hormonal effect D. Pregnancy

The correct answer is D Pregnancy must be excluded in any woman of reproductive age before considering other diagnoses. Although this patient is perimenopausal—by her age and irregular menses—she is not yet menopausal, which requires 12 months of amenorrhea in this age range. Menopause is the second most common time for unplanned pregnancy. Hormonal effect is possible, especially if she was taking hormonal therapy, but would be assessed only after excluding pregnancy. Dysfunctional uterine bleeding is a diagnosis of exclusion that is unlikely to play a role in this patient's presentation of isolated perimenopausal bleeding.

You evaluate a 52-year-old nurse who lost consciousness while reading in bed. Her husband witnessed the episode and phoned 911. Which historical feature is most concerning for a serious cause of syncope? A. Loss of consciousness B. Spontaneous, prompt, and complete recovery C. Sweating and nausea D. Occurrence in the supine position

The correct answer is D The first 3 options are typical of vasovagal syncope. Loss of consciousness in the supine position is an alarm symptom suggesting a cardiac cause such as an arrhythmia.

A 35-year-old man presents with fever and pain behind his left eye. One week earlier, he had a "cold" with rhinorrhea and nasal congestion that progressed to a "sinus headache." His nasal congestion worsened over the past few days, and he developed left-sided tooth pain and pressure behind his left eye. Over the past 24 hours, he reports swelling around the eye, diffuse redness, fever of 101°F, pain with eye movement, and mild diplopia. He denies discharge, photophobia, or foreign body sensation. Which of the following is the most likely diagnosis? A. Episcleritis B. Bacterial conjunctivitis C. Periorbital cellulitis D. Orbital cellulitis

The correct answer is D The patient has sinusitis and is now presenting with symptoms consistent with orbital cellulitis: fevers, diffuse erythema, diplopia, and pain with eye movement secondary to inflammation of the tissues/muscles in the orbit. Periorbital cellulitis would also cause fevers and swelling but would not result in diplopia because the inflammation is anterior to the orbital septum.

You see a 55-year-old woman in the office who reports intermittent chest pain for the last 3 months. She has continued to work as a medical coder but has stopped exercising since she noticed these pains. Which of the following features would suggest that her chest pain is not related to myocardial ischemia? A. Relationship to effort B. Squeezing quality C. Association with dyspnea D. Pain is sharp and increases with inspiration

The correct answer is D The squeezing, oppressive quality, precipitation by exertion, and associated dyspnea are all consistent with ischemic chest pain. Sharp or stabbing pain, pleuritic pain, localized pain, positional pain, and chest pain reproduced by palpation all decrease the likelihood that the symptoms are secondary to myocardial ischemia.

A 25-year-old medical student agrees to have 75 mL of blood drawn for a scientific study. While the phlebotomist is attempting to draw the blood, she encounters difficulty and begins manipulating the needle and probing. The medical student becomes pale, cold, and clammy. A few seconds later, he slumps over in the chair and has several brief jerking movements of the arms and legs. He is helped down to the floor and promptly regains consciousness. After a few brief moments of confusion, he appears normal and is quite embarrassed. What is the most likely diagnosis? A. Seizure B. Orthostatic hypotension C. Carotid sinus syncope D. Vasovagal syncope

The correct answer is D This is a typical story for vasovagal syncope or, in this case, situational syncope, which is triggered by a painful stimulus. Brief jerking movements can be seen in all forms of syncope and must be differentiated from the tonic-clonic movements of a seizure. The lack of tongue biting or postictal confusion makes seizure less likely. Removal of 75 mL of blood is insufficient to cause orthostatic hypotension. Finally, there is no antecedent, reproducible history of neck turning or a tight collar to suggest carotid sinus syncope.

A 25-year-old woman with a history of 3 miscarriages and no live births presents with worsening pelvic pain over the last 4 months. Her last pregnancy was 2 years ago. She has had dysmenorrhea since she began her periods at 15 years. She has noticed dyspareunia over the last year. Her menstrual periods are irregular, but her flow is normal. Her last period was 1 week ago. She has been taking fluoxetine for depression and anxiety for 3 months. She denies any fever, chills, or vaginal discharge. What is the most likely diagnosis? A. Pelvic inflammatory disease B. Ectopic pregnancy C. Adnexal torsion D. Endometriosis

The correct answer is D This patient has chronic pelvic pain, which virtually rules out causes of acute pelvic pain like ectopic pregnancy, pelvic inflammatory disease, and adnexal torsion. The history of infertility, deep dyspareunia, and irregular menses is characteristic of endometriosis. Absence of fever and vaginal discharge makes pelvic inflammatory disease unlikely, and recent normal period makes pregnancy-related causes like ectopic pregnancy unlikely.

A 78-year-old man is brought in by his daughter who has noticed bruises on her father. He is falling and bumping into objects around him. He is well-known to you and has a history of long-standing diabetes mellitus and remote alcohol abuse. When queried, he states, "I get a little dizzy sometimes and slip." He has problems with his vision and also numbness and tingling in his feet. He is hard of hearing. This form of dizziness can be classified as: A. Vertigo B. Presyncope C. Light-headedness D. Disequilibrium

The correct answer is D This patient has disequilibrium from the syndrome of multisensory deficits. Peripheral neuropathy from long-standing diabetes mellitus and prior alcohol abuse would decrease his touch and proprioceptive abilities. Additionally, his vision is impaired and he has hearing loss. He does not report any vertiginous symptoms, such as spinning or tilting. The patient does not describe any orthostatic symptoms.

You are seeing an 80-year-old man with a 2-day history of dysuria and increased urinary frequency. These symptoms have worsened over the last day, and this morning, he was unable to urinate. He also developed fever, shaking chills, and lower abdominal pain. He underwent bladder catheterization during a hospitalization 1 week ago. He denies back pain, hematuria, and scrotal swelling or pain. What is the most likely diagnosis? A. Prostate cancer B. Bacterial epididymitis C. Bladder outlet obstruction from benign prostatic hyperplasia D. Acute bacterial prostatitis

The correct answer is D This patient has new-onset dysuria and urinary frequency suggesting bladder irritation or infection. His worsening voiding symptoms are concerning for acute urinary retention. This constellation of symptoms, along with systemic symptoms suggesting acute infection, makes acute bacterial prostatitis the most likely diagnosis. Benign prostatic hyperplasia and recent bladder catheterization are the 2 most common risk factors for acute prostatitis. Although epididymitis can also cause fever and chills and may be seen in patients with bacterial prostatitis, this patient's lack of scrotal swelling and pain make epididymitis less likely. His symptoms are concerning for acute bladder outlet obstruction, which is likely due to a swollen, inflamed prostate rather than benign prostatic hyperplasia alone.

You have just diagnosed bacterial urinary tract infection (UTI) in a 56-year-old woman. She reports 3 UTIs in the past year and is concerned about why they keep recurring. She has multiple sclerosis and was diagnosed with type 2 diabetes 1 year ago. She has chronic, intermittent urinary incontinence, which is large volume and occurs without warning in both the daytime and at night. She denies vaginal dryness, dyspareunia, hematuria, or unilateral back pain but had a kidney stone 10 years earlier. What is the most likely explanation for her recurrent urinary tract infections? A. Retained kidney stone B. Diabetes C. Atrophic vaginitis D. Chronic incontinence

The correct answer is D This patient has several risk factors for recurrent UTI. Chronic incontinence increases the risk of recurrent UTI in postmenopausal women. Her symptoms of incontinence also suggest neurogenic bladder, due to multiple sclerosis. With neurogenic bladder, incomplete bladder emptying and retained urine can lead to recurrent UTIs. Although atrophic vaginitis is also a risk factor for UTI in postmenopausal women, this patient does not have symptoms of vaginal dryness or pain with intercourse, making it a less likely explanation. Although a retained kidney stone could serve as a nidus for recurrent infections, her lack of hematuria, flank pain, or symptoms of nephrolithiasis make this an unlikely explanation for her recurrent UTIs.

A 26-year-old woman presents with sore throat, odynophagia, and fevers for 8 days. Over the last 24 hours, she has been unable to swallow her secretions and is drooling. She has trouble opening her mouth and has noted that her anterior neck seems swollen. Which of the following should you consider? A. Acute epiglottitis B. Retropharyngeal abscess C. Lemierre syndrome D. All of the above

The correct answer is D This patient presents with alarm symptoms including drooling, trismus, and neck swelling, suggesting a severe suppurative complication of acute pharyngitis. The causative agent could be GAS, F necrophorum, or another anaerobic or streptococcal infection, all of which can cause these symptoms. Emergent ear, nose, and throat evaluation and neck imaging are likely the next steps in the evaluation.


Conjuntos de estudio relacionados

Hazing Prevention 101 Course - College Edition 2021

View Set

Geometric Series and Applications Std.A.SSE.4

View Set

Chapter 23 - Distributed Database Concepts

View Set

Chapter 15- The Kennedy & Johnson Years

View Set

Ch. 15: Regulation of Gene Expression

View Set

WGU study guide for care of the older adult OA

View Set

OMM Semester 1 Savarese Practice Comlex Exam

View Set